curso de matematica olimpica poti

1198
Curso de Treinamento em Matemática Olímpica Compilação Omegaleph dos Materiais dos Cursos dos Polos de Treinamento Olímpico POTI-IMPA Teoria dos Números Álgebra Combinatória Geometria 25 de Dezembro de 2013

Upload: filipe-miguel

Post on 19-Jan-2016

624 views

Category:

Documents


21 download

DESCRIPTION

Curso de Matematica

TRANSCRIPT

  • Curso de Treinamento em Matemtica Olmpica

    Compilao Omegaleph dos Materiais dos Cursos dos Polos de Treinamento Olmpico

    POTI-IMPA

    Teoria dos Nmeros lgebra

    Combinatria Geometria

    25 de Dezembro de 2013

  • Polos Olmpicos de TreinamentoCurso de Teoria dos Nmeros - Nvel 2Samuel Barbosa Feitosa

    Aula 1

    Divisibilidade I

    Teorema 1. (Algoritmo da Divisao) Para quaisquer inteiros positivos a e b, existe um unicopar (q, r) de inteiros nao negativos tais que b = aq + r e r < a. Os numeros q e r saochamados de quociente e resto, respectivamente, da divisao de b por a.

    Exemplo 2. Encontre um numero natural N que, ao ser dividido por 10, deixa resto 9, aoser dividido por 9 deixa resto 8, e ao ser dividido por 8 deixa resto 7.

    O que acontece ao somarmos 1 ao nosso numero? Ele passa a deixar resto 0 na divisao por10, 9 e 8. Assim, um possvel valor para N e 10 9 8 1.

    Exemplo 3. a) Verifique que an 1 = (a 1)(an1 + an2 + . . .+ a+ 1)

    b) Calcule o resto da divisao de 42012 por 3.

    Para o item a), usando a distributividade e efetuando os devidos cancelamentos no ladodireito, podemos escrever:

    an + an1 + . . .+ a2 + a an1 an2 . . . a 1 = an 1.

    Para o item b), veja que 3 = 41 e assim e natural substituir os valores dados na expressaodo primeiro item:

    42012 1 = 3(42011 + . . .+ 4 + 1).

    Isso significa que q = (42011 + . . .+ 4 + 1) e que r = 1.

    Observacao 4. O teorema anterior admite um enunciado mais geral: Para quaisquer intei-ros a e b, com a 6= 0, existe um unico par de inteiros (q, r) tais que b = aq+ r, 0 r < |a|.Por exemplo, o resto da divisao de 7 por 3 e 2 e o quociente e 3.

    Iremos agora estudar propriedades a respeito das operacoes com restos.

    Teorema 5. (Teorema dos Restos) Se b1 e b2 deixam restos r1 e r2 na divisao por a,respectivamente,entao:

  • POT 2012 - Teoria dos Numeros - Nvel 2 - Aula 1 - Samuel Feitosa

    b1 + b2 deixa o mesmo resto que r1 + r2 na divisao por ab1b2 deixa o mesmo resto que r1r2 na divisao por a.

    Demonstracao. Por hipotese, existem q1, q2 e q tais que: b1 = aq1 + r1, b2 = aq2 + r2 er1 + r2 = aq + r, logo:

    b1 + b2 = a(q1 + q2 + q) + r.

    Como 0 < r < |a|, b1 + b2 deixa resto r quando dividido por a. A demonstracao para oproduto e deixada ao cargo do leitor.

    Observacao 6. Em alguns casos, e prefervel que o professor faca uma demonstracao doresultado anterior para a = 3 ou a = 5 apenas com o intuito de deixar os alunos maisconfortaveis a respeito do resultado. E prefervel que mais tempo seja gasto resolvendoexemplos e problemas. Na secao de congruencias, os alunos terao um contato mais apro-priado com o enunciado anterior.

    Exemplo 7. Qual o resto que o numero 1002 1003 1004 deixa quando dividido por 7?

    Como 1002 deixa resto 1 por 7, o numero acima deixa o mesmo resto que 1 2 3 = 6 por 7.

    Exemplo 8. Qual o resto que o numero 45000 deixa quando dividido por 3?

    Como 4 deixa resto 1 por 3, 45000 deixa o mesmo resto que 1 1 . . . 1 5000

    = 1 por 3.

    Exemplo 9. Qual o resto que o numero 22k+1 deixa quando dividido por 3?

    Note que 20 deixa resto 1 por 3, 21 deixa resto 2 por 3, 22 deixa resto 1 por 3, 23 deixaresto 2 por 3, 24 deixa resto 1 por 3. Precebeu alguma coisa? Como 100 e par, o restodevera ser 1. Como 22 deixa resto 1, entao 22k = 22 22 . . . 22

    k

    deixa o mesmo resto que

    1 1 . . . 1 k

    = 1 e 22k+1 = 22k 2 deixa o mesmo resto que 1 2 = 2 por 3.

    Exemplo 10. Qual o resto de n3 + 2n na divisao por 3?

    Se o resto de n por 3 e r, o resto de n3 + 2n e o mesmo de r3 + 2r. Para r = 0, esseresto seria 0. Para r = 1, seria o mesmo resto de 3 que e 0. Finalmente, para r = 2, oresto seria o mesmo de 8 + 4 = 12 que tambem e 0. Assim, nao importa qual o resto de npor 3, o numero n3 + 2n sempre deixara resto 0. Uma ideia importante nessa solucao foidivid-la em casos. Tambem poderamos ter resolvido esse exemplo apelando para algumafatoracao:

    n3 + 2n = n3 n+ 3n = n(n2 1) + 3n = n(n 1)(n+ 1) + 3n.

    Como n1, n e n+1 sao consecutivos, um deles e multiplo de 3. Assim, o ultimo termo daigualdade anterior e a soma de dois multiplos de 3 e consequentemente o resto procuradoe 0.

    2

  • POT 2012 - Teoria dos Numeros - Nvel 2 - Aula 1 - Samuel Feitosa

    Observacao 11. Fatoracoes podem ser muito uteis para encontrarmos os valores explcitosde q e r.

    Exemplo 12. Prove que, para cada n natural,

    (n+ 1)(n+ 2) . . . (2n)

    e divisvel por 2n.

    Veja que

    (n+ 1)(n+ 2) . . . (2n) =1 2 2n

    1 2 n.

    Para cada numero natural k no produto escrito no denominador, temos uma aparicao de2k no produto escrito no numerador. Basta efetuarmos os cancelamentos obtendo:

    (n+ 1)(n+ 2) . . . (2n) = 2n 1 3 (2n 1).

    Exemplo 13. (Olimpada de Leningrado 1991) Cada um dos naturais a, b, c e d e divisvelpor ab cd, que tambem e um numero natural. Prove que ab cd = 1.

    Se chamarmos p = ab cd, teremos a = px, b = py, c = pz e d = pt onde x, y, z e t saointeiros. Assim, p = p2(xyzt). Consequentemente 1 = p(xyzt) e conclumos que p = 1,pois p e natural.

    Exemplo 14. A soma digital D(n) de um inteiro positivo n e definida recursivamente comosegue:

    D(n) =

    {n se 1 n 9,D(a0 + a1 + . . .+ am) se n > 9,

    onde a0, a1, . . . , am sao todos os dgitos da expressao decimal de n na base 10, i.e.,

    n = am10m + am110

    m1 + . . .+ a110 + a0

    Por exemplo, D(989) = D(26) = D(8) = 8. Prove que: D((1234)n) = D(n), para n =1, 2, 3 . . .

    Como 10n1n = (101)(10n1+10n2+ . . .+1), podemos concluir que 10n sempre deixaresto 1 na divisao por 9. Assim, n = am10

    m+am110m1+ . . .+a110+a0, deixa o mesmo

    resto que am + am1 + . . . + a0 na divisao por 9. Desse modo, D(n) nada mais e do queo resto na divisao por 9 do numero n. Como 1234 deixa resto 1 por 9, o numero (1234)ndeixa o mesmo resto que 1 n por 9, ou seja, D((1234)n) = D(n).

    Observacao 15. O exemplo anterior contem o criterio de divisibilidade por 9, i.e., n deixao mesmo resto que D(n) na divisao por 9. O criterio de divisibilidade por 3 e analogo pois10n tambem sempre deixa resto 1 por 3.

    Exemplo 16. Encontre todos os pares de inteiros positivos a e b tais que 79 = ab+2a+3b.

    3

  • POT 2012 - Teoria dos Numeros - Nvel 2 - Aula 1 - Samuel Feitosa

    Fatoremos a expressao anterior. Somando 6 aos dois lados da equacao, obtemos:

    85 = 6 + ab+ 2a+ 3b

    = (3 + a)(2 + b)

    Assim, (3 + a) e (2 + b) sao divisores positivos de 85 maiores que 1. Os unicos divisorespositivos de 85 sao 1, 5, 19, 85. Logo, os possveis pares de valores para (3 + a, 2 + b) sao(5, 19) ou (19, 5) que produzem as solucoes (a, b) = (2, 17) e (16, 3).

    Problema 17. (Olimpada Russa) Prove que se2n 2

    ne um inteiro, entao

    22n1 2

    2n 1tambem e um inteiro.

    Se k =2n 2

    n, entao

    22n1 2

    2n 1=

    2(22n2 1)

    2n 1

    = 2

    (2nk 1

    2n 1

    )

    = 2

    ((2n 1)(2n(k1) + 2n(k2) + . . .+ 2n + 1)

    2n 1

    )

    = 2(2n(k1) + 2n(k2) + . . .+ 2n + 1),

    e um numero inteiro.

    Problemas Propostos

    Problema 18. Encontre os inteiros que, na divisao por 7, deixam um quociente igual aoresto.

    Problema 19. Determinar os numeros que divididos por 17 dao um resto igual ao quadradodo quociente correspondente.

    Problema 20. (OCM 1985) Encontre o quociente da divisao de a128 b128 por

    (a64 + b64)(a32 + b32)(a16 + b16)(a8 + b8)(a4 + b4)(a2 + b2)(a+ b)

    Problema 21. (OCM 1994) Seja A = 777 . . . 77 um numero onde o dgito 7aparece 1001vezes. Determinar o quociente e o resto da divisao de A por 1001.

    Problema 22. Encontre um inteiro que deixa resto 4 na divisao por 5 e resto 7 na divisaopor 13

    Problema 23. Encontre o menor inteiro que, dividido por 29 deixa resto 5, e dividido por31 da resto 28.

    4

  • POT 2012 - Teoria dos Numeros - Nvel 2 - Aula 1 - Samuel Feitosa

    Problema 24. Prove que, para todo inteiro positivo n o numero n5 5n3 + 4n e divisvelpor 120.

    Problema 25. (Fatoracoes Importantes)

    a) Seja S = 1+ z+ z2+ z3+ . . .+ zn1. Veja que S+ zn = 1+ zS entao S(z1) = zn1.Conclua que, para quaisquer x e y vale:

    xn yn = (x y)(xn1 + xn2y + xn3y2 + . . .+ x2yn3 + xyn2 + yn1)

    b) Mostre que se n e mpar vale:

    xn + yn = (x+ y)(xn1 xn2y + xn3y2 . . .+ x2yn3 xyn2 + yn1)

    Problema 26. Prove que, o numero 199 + 299 + 399 + 499 + 599 e multiplo de 5.

    Problema 27. Mostre que o numero 1n + 8n 3n 6n e multiplo de 10 para todo naturaln.

    Problema 28. Encontre o resto da divisao 3710 1 por 11.

    Problema 29. Prove que 22225555 + 55552222 e divisvel por 7.

    Problema 30. Encontre o ultimo dgito do numero 19891989.

    Problema 31. Mostre que se n divide a entao 2n 1 divide 2a 1.

    Problema 32. (Cone Sul 1996) Provar que o numero

    1995 19971996 1996 19971995 + 1

    19962

    e um inteiro.

    Problema 33. Mostre que para n mpar, n divide 1n + 2n + . . .+ (n 1)n

    Problema 34. Existe um natural n tal que nn + (n+ 1)n e divisvel por 2011?

    Problema 35. Quantos numeros inteiros positivos n existem tais que n+ 3 divide n2 + 7?

    Problema 36. Encontre o numero de inteiros n tais que

    1. 1000 < n < 8000.

    2. nn+1 + (n+ 1)n e divisvel por 3.

    Problema 37. Sejam m e n naturais tais que mn+ 1 e multiplo de 24, mostre que m+ ntambem e multiplo de 24.

    5

  • POT 2012 - Teoria dos Numeros - Nvel 2 - Aula 1 - Samuel Feitosa

    Problema 38. (Irlanda 1997) Encontre todos os pares de inteiros (x, y) tais que 1+1996x+1998y = xy.

    Dicas e Solucoes

    18. Os numeros sao {0, 8, 16, 24, . . . , 8 7}.

    18. Escreva n = 17q + q2 e note que 0 q2 < 17. Assim, q = 0, 1, 2, 3, 4.

    19. Use a diferenca de quadrados sucessivas vezes para obter (a b) como quociente.

    21. O numero do problema e igual a 7(101001

    1)9 . Alem disso,

    10999+1103+1

    e inteiro e 101001

    1103+1

    =

    100 10999+1

    103+1 100

    103+1.

    22. Os numeros que satisfazem essa propriedade sao os numeros da forma 65k + 59.

    24. Basta mostrar que n5 5n3 + 4n e multiplo de 3, 8 e 5. Na divisao por 5, temosquatro restos possveis: {0, 1, 2, 3, 4}. Assim, o numero n55n3+4n possui o mesmoresto na divisao por 5 que um dos cinco numeros: {05 5 03+40, 15 5 13+4, 255 23 + 8, 35 5 33 + 12, 45 5 43 + 16}. Como todos esses numeros sao multiplosde 5, segue que n5 5n3 + 4n e multiplo de 5 para todo n inteiro. O procedimentocom 3 e 8 e semelhante.

    25. Para o item a), troque z por xy. Para o item b), substitua y por y no item anterior.

    26. Pelo problema anterior, como 99 e mpar temos: 199 + 499 = (1 + 4)(198 + 197 4 +. . .+ 1 497 + 498). Da, segue que 199 + 499 e multiplo de 5. Analogamente podemosmostrar que 299 + 399 e multiplo de 5.

    27. O numero em questao e mutiplo de 2 pois e a soma de dois mpares e dois pares.Para ver que tambem e multiplo de 5, basta notar que 5 divide 1n 6n e 8n 3n.Isso pode ser facilmente mostrado usando a fatoracao do exerccio 25.

    31. Se a = nk, temos (2n 1)(2n(k1) + 2n(k2) + . . .+ 2n + 1) = 2nk 1.

    32. Veja que 199519971996199619971995+1 = 1995(199719961)1996(199719951).Pela fatoracao de xn yn,

    1996 (19971995 1)

    19962= (19971994 + 19971993 + . . .+ 1),

    e inteiro. Alem disso, pela mesma fatoracao,

    1995 (19971996 1)

    19962= 1995

    (19971995 1

    1996+

    19971994 1

    1996+ . . .+

    1997 1

    1996+

    1996

    1996

    ),

    e uma soma de numeros inteiros.

    6

  • 33. Como n e impar,

    (n i)n + in = ((n i) + i)((n i)n1 (n i)n2i+ . . . (n i)in2 + in1).

    34. Faca n = 1005 e use a fatoracao de xn + yn.

    37. Fatore a expressao como:

    (x 1998)(y 1996) = xy 1998y 1996x+ 1998 1996 = 19972.

    Os divisores de 19972 sao {1,1997,19972}. Resolvendo os sistemas correspon-dentes a` essas possibilidades, temos: (x, y) = (1999, 19972 + 1996), (1997,19972 +1996), (3995, 3993), (1,1), (19972 + 1998, 1997), (19972 + 1998, 1995).

    Referencias

    [1] F. E. Brochero Martinez, C. G. Moreira, N. C. Saldanha, E. Tengan - Teoria dosNumeros um passeio com primos e outros numeros familiares pelo mundo inteiro,Projeto Euclides, IMPA, 2010.

    [2] E. Carneiro, O. Campos and F. Paiva, Olimpadas Cearenses de Matematica 1981-2005(Nveis Junior e Senior), Ed. Realce, 2005.

    [3] S. B. Feitosa, B. Holanda, Y. Lima and C. T. Magalhaes, Treinamento Cone Sul 2008.Fortaleza, Ed. Realce, 2010.

    [4] D. Fomin, A. Kirichenko, Leningrad Mathematical Olympiads 1987-1991, MathProPress, Westford, MA, 1994.

    [5] D. Fomin, S. Genkin and I. Itenberg, Mathematical Circles, Mathematical Words, Vol.7, American Mathematical Society, Boston, MA, 1966.

    [6] I. Niven, H. S. Zuckerman, and H. L. Montgomery, An Introduction to the Theory ofNumbers.

  • Polos Olmpicos de TreinamentoCurso de Teoria dos Nmeros - Nvel 2Prof. Samuel Feitosa

    Aula 2

    Divisibilidade II

    Definicao 1. Dados dois inteiros a e b, com a 6= 0, dizemos que a divide b ou que a eum divisor de b ou ainda que b e um multiplo de a e escrevemos a | b se o r obtido peloalgoritmo de divisao aplicado a` a e b e 0, ou seja, se b = aq para algum inteiro q.

    Lema 2. Sejam a, b, c, d inteiros. Temos

    i) (d divide) Se d | a e d | b, entao d | ax+ by para quaisquer x e y inteiros.ii) (Limitacao) Se d | a, entao a = 0 ou |d| |a|.iii) (Transitividade) Se a | b e b | c, entao a | c.Em particular, segue da propriedade i) que d | a+ b e d | a b.Exemplo 3. (Olimpada de Maio 2006) Encontre todos os naturais a e b tais que a|b+ 1 eb|a+ 1.Pela propriedade da Limitacao, temos a b + 1 e b a + 1. Da, a 1 b a + 1.Vejamos os casos:

    (i) a = b. Como a|b+ 1 e a | b(pois b = a) temos que a | [(b+ 1) b] = 1. Assim, a = 1Nesse caso, so temos a solucao (a, b) = (1, 1)

    (ii) a = b+ 1. Como b|a+ 1 e b|a 1(pois b = a 1) temos que b|[(a+ 1) (a 1)] = 2.Assim, b = 1 ou b = 2 e nesse caso, so temos as solucoes (3, 2) e (2, 1).

    (iii) a = b 1. Esse caso e analogo ao anterior e as solucoes para (a, b) sao (1, 2) e (2, 3).Exemplo 4. (Criterio de Divisibilidade por 7) Existem alguns metodos praticos para deci-dirmos se um numero e multiplo de outro. Certamente o leitor ja deve ter se deparado comalgum criterio de divisibilidade. Existe um criterio por 7 bastante popular: Para saber seum inteiro e multiplo de 7, basta apagar seu ultimo dgito, multiplica-lo por 2 e o subtrairdo numero que restou. Se o resultado e multiplo de 7, entao o numero original tambem emultiplo de 7.

  • POT 2012 - Teoria dos Numeros - Nvel 2 - Aula 1 - Samuel Feitosa

    Podemos aplicar esse algoritmo sucessivas vezes ate que o resultado obtido seja facil-mente verificavel como um multiplo de 7. Por exemplo, para o numero 561421 podemosescrever:

    56142 2 = 561405614 0 = 5614561 8 = 55355 6 = 49

    Como 49 e multiplo de 7, nosso numero original tambem e. Por que esse processo funciona?Se o nosso numero original esta escrito na forma 10a + b, entao o numero obtido apos aoperacao descrita e a 2b. Basta mostrarmos que se 7 | a 2b, entao 7 | 10a + b. Se7 | a 2b, pela propriedade (i) do lema, conclumos que 7 | 10a 20b. Como 7 | 21b,tambem temos que 7 | [(10a 20b) + 21b] = 10a+ b.Exemplo 5. Mostre que se 7 | 3a+ 2b entao 7 | 4a 2b.Veja que 7 | 7a e 7 | 3a + 2b, entao 7 | [7a (3a + 2b)] = 4a 2b. Na pratica, o quefizemos foi multiplicar o numero 3a+2b por algum inteiro para posteriormente subtramosum multiplo de 7 conveniente e obtermos o numero 4a 2b. Existem outras formas defazermos isso. Observe os numeros 3 0, 3 1, 3 2, 3 3, 3 4, 3 5, 3 6. O numero 3 6 deixao mesmo resto que 4 por 7, pois 3 6 = 7 2 + 4. Como 7|3a + 2b podemos concluir que7|(18a + 12b) e consequentemente 7 | [18a + 12b 14a)] = 4a + 12b. Mas 7 | 14b, entao7 | [4a+ 12b 14b] = 4a 2b.

    Para o proximo exemplo, o leitor precisara lembrar dos criterios de divisibilidade por 9e 3 vistos na aula passada.

    Exemplo 6. Usando os dgitos 1, 2, 3, 4, 5, 6, 7, construmos varios numeros de sete dgitosdistintos. Existem dois deles, distintos, tais que um divide o outro?

    Nao. Suponha, por absurdo, que m < n sejam dois desses numeros, com m | n. Claramentem | nm e 9 | nm, pois n e m possuem a mesma soma dos dgitos e consequentementepossuem o mesmo resto na divisao por 9. Por outro lado, sabemos a soma dos dgitos dem: 1+2+ +7 = 3 9+1. Da, m nao possui fator 9 e podemos garantir que 9m | nm.Mas entao 9m nm 10m n n tem pelo menos oito dgitos, uma contradicao.

    Exemplo 7. (Leningrado 1989) Seja A um numero natural maior que 1, e seja B umnumero natural que e um divisor de A2 + 1. Prove que se B A > 0, entao B A > A.Seja B A = q. Assim, A + q | A2 + 1. Como (A q)(A + q) = A2 q2 e divisvel porA+ q, podemos concluir que A+ q | [(A2 + 1) (A2 q2)] = q2 + 1. Pela propriedade delimitacao, A+ q q2 +1. Nessa desigualdade, nao podemos ter q = 1 pois A > 1. Usandoentao que q > 1, temos A q2 q + 1 < q2, ou seja, A < q.Problema 8. (AIME 1986) Qual e o maior inteiro n para o qual n3 + 100 e divisvel porn+ 10?

    2

  • POT 2012 - Teoria dos Numeros - Nvel 2 - Aula 1 - Samuel Feitosa

    Para achar explicitamente o quociente de n3+100 por n+10 podemos fazer uso de algumafatoracao. Utilizaremos a soma dos cubos n3 + 103 = (n+ 10)(n2 10n+ 100). Como,

    n3 + 100 = (n+ 10)(n2 10n+ 100) 900,

    podemos concluir que o numero 900 deve ser multiplo de n+ 10. O maior inteiro n para oqual n+ 10 divide 900 e 890. Veja que se n = 890, o quociente da divisao de n3 + 100 porn+ 10 e n2 10n+ 100 1 = 8902 10 890 + 99.Exemplo 9. (Extrado de [1]) Encontre todos os inteiros positivos n tais que 2n2 + 1 |n3 + 9n 17.Utilizando o 2n2 + 1 divide para reduzir o grau de n3 + 9n 17, temos que{

    2n2 + 1 | n3 + 9n 172n2 + 1 | 2n2 + 1

    = 2n2 + 1 | (n3 + 9n 17) 2 + (2n2 + 1) (n) 2n2 + 1 | 17n 34

    Como o grau de 17n 34 e menor do que o de 2n2 + 1, podemos utilizar a limitacaopara obter uma lista finita de candidatos a n. Temos 17n 34 = 0 n = 2 ou|2n2 + 1| |17n 34| n = 1, 4 ou 5. Destes candidatos, apenas n = 2 e n = 5 saosolucoes.

    Exemplo 10. (Leningrado 1990) Sejam a e b numeros naturais tais que b2 + ba+ 1 dividea2 + ab+ 1. Prove que a = b.

    Pela propriedade de limitacao, b2+ba+1 a2+ab+1 e da b a. Alem disso, b2+ab+1 >a b. A igualdade b(a2 + ab+ 1) a(b2 + ba+ 1) = b a implica que a b e divisvel porb2+ ba+1. Se a b 6= 0, entao b2+ ab+1 a b. Mas isso e um absurdo, logo a b = 0.

    Problemas Propostos

    Problema 11. Mostre que se 3 | a+ 7b entao 3 | a+ b.Problema 12. Mostre que se 7 | a+ 3b entao | 13a+ 11bProblema 13. Mostre que se 19 | 3x+ 7y entao 19 | 43x+ 75yProblema 14. Mostre que se 17 | 3a+ 2b entao 17 | 10a+ bProblema 15. Encontre todos os inteiros positivos n tais que n+ 2009 divide n2 + 2009 en+ 2010 divide n2 + 2010.

    Problema 16. Seja n > 1 e k um inteiro positivo qualquer. Prove que (n 1)2|(nk 1) se,e somente se , (n 1)|k.

    3

  • POT 2012 - Teoria dos Numeros - Nvel 2 - Aula 1 - Samuel Feitosa

    Problema 17. (OBM 2005) Prove que a soma 1k + 2k + . . .+ nk, onde n e um inteiro e ke mpar, e divisvel por 1 + 2 + . . .+ n.

    Problema 18. O numero de seis dgitos X = abcdef satisfaz a propriedade de que abcdefe divisvel por 7. Prove que X tambem e divisvel por 7.

    Problema 19. (Bielorussia 1996) Inteiros m e n, satisfazem a igualdade

    (m n)2 = 4mnm+ n 1 .

    a) Prove que m+ n e um quadrado perfeito.

    b) Encontre todos os pares (m,n) satisfazendo a equacao acima.

    Problema 20. (Olimpada de Leningrado) Os numeros naturais a,b e c tem a propriedadeque a3 e divisvel por b, b3 e divisvel por c e c3 e divisvel por a. Prove que (a+ b+ c)13 edivisvel por abc.

    Problema 21. (OBM 2000) E possvel encontrar duas potencias de 2, distintas e com omesmo numero de algarismos, tais que uma possa ser obtida atraves de uma reordenacaodos dgitos da outra? (Dica: Lembre-se do criterio de divisibilidade por 9)

    Problema 22. (IMO 1998) Determine todos os pares de inteiros positivos (x, y) tais quexy2 + y + 7 divide x2y + x+ y.

    Dicas e Solucoes

    11. Como 3 | 6b, segue que 3 | [(a+ 7b) 6b] = a+ b.12. Como 7 | a+ 3b, segue que 7 | 13a+ 39b = (13a+ 11b) + 28b. Mas 7 | 28b, portanto

    7 | [(13a+ 11b) + 28b 28b] = 13a+ 11b.13. Como 19 | 3x + 7y, segue que 19 | 27(3x + 7y) = (43x + 75y) + (38x + 114y). Mas

    19 | 19(2x+6y), portanto 19 | [(43x+75y)+(38x+114y)19(2x+6y)] = 43x+75y.14. Como 17 | 3a+ 2b, segue que 17 | 27a+ 18b = (10a+ b) + 17(a+ b).16. Veja que

    nk 1(n 1)2 =

    (nk1 1n 1 +

    nk2 1n 1 + . . .+

    n 1n 1 +

    k

    n 1).

    Como os numeros nl1

    n1 sempre sao inteiros, o numero do lado esquerdo da equacao

    sera inteiro se, e somente se, o numero kn1 for inteiro.

    4

  • POT 2012 - Teoria dos Numeros - Nvel 2 - Aula 1 - Samuel Feitosa

    17. Comece dividindo o problema quando em dois casos: n e par ou n e mpar. Sabemosque 1+ 2+ . . .+ n = n(n+1)2 . Para n mpar, basta mostrar que o numero em questaoe divisvel por n e n+12 . O proximo passo e lembrar do problema 33 da aula 1. Pelafatoracao de xn + yn, temos que ik + (n i)k e divisvel por n. Faca outros tipos depares para mostrar a divisibilidade por n2 . O caso quando n e par e analogo.

    18. Veja que X = 103 abc + def = 1001abc (abc def). Como 1001 e multiplo de 7,conclumos que X e a soma de dois multiplos de 7.

    19. Somando 4mn em ambos os lados, obtemos:

    (m+ n)2 =4mn

    m+ n 1 + 4mn

    =4mn(m+ n)

    m+ n 1

    (m+ n) =4mn

    m+ n 1= (m n)2.

    Assim, m + n e o quadrado de um inteiro. Se m n = t, entao m + n = t2 e(m,n) = ( t

    2+t2 ,

    t2t

    2 ). E facil verificar que para qualquer t inteiro esse par e solucaodo problema.

    20. Analise a expansao pelo binomio de Newton.

    21. Nao. Suponha, por absurdo, que existam duas potencias de 2, 2m < 2n, satisfazendoo enunciado. Como 2n e um multiplo de 2m, podemos ter: 2n = 22m, 42m, 82m, . . ..Alem disso, como ambos possuem a mesma quantidade de dgitos, temos 1 < 2

    n

    2m 0 e divisvel por xy2 + y + 7. Da, xy2 + y + 7 7x y2 < 7x, quenos permite concluir que y 2. Para y = 1, temos x+8 | 7x 1 e consequentementex+ 8 | 7(x+ 8) (7x 1) = 57. Entao as unicas possibilidades sao x = 11 e x = 49,cujos pares correspondentes sao (11, 1), (49, 1). Para y = 2, temos 4x + 9 | 7x 4e consequentemente 7(4x + 9) 4(7x 4) = 79 e divisvel por 4x + 9. Nesse caso,nao obtemos nenhuma solucao nova. Todas as solucoes para (x, y) sao: (7t2, 7t)(t N), (11, 1) e (49, 1).

    5

  • Referencias

    [1] F. E. Brochero Martinez, C. G. Moreira, N. C. Saldanha, E. Tengan - Teoria dosNumeros um passeio com primos e outros numeros familiares pelo mundo inteiro,Projeto Euclides, IMPA, 2010.

    [2] E. Carneiro, O. Campos and F. Paiva, Olimpadas Cearenses de Matematica 1981-2005(Nveis Junior e Senior), Ed. Realce, 2005.

    [3] S. B. Feitosa, B. Holanda, Y. Lima and C. T. Magalhaes, Treinamento Cone Sul 2008.Fortaleza, Ed. Realce, 2010.

    [4] D. Fomin, A. Kirichenko, Leningrad Mathematical Olympiads 1987-1991, MathProPress, Westford, MA, 1994.

    [5] D. Fomin, S. Genkin and I. Itenberg, Mathematical Circles, Mathematical Words, Vol.7, American Mathematical Society, Boston, MA, 1966.

    [6] I. Niven, H. S. Zuckerman, and H. L. Montgomery, An Introduction to the Theory ofNumbers.

  • Polos Olmpicos de TreinamentoCurso de Teoria dos Nmeros - Nvel 2Prof. Samuel Feitosa

    Aula 3

    O Algoritmo de Euclides

    Exemplo 1. Seja S um conjunto infinito de inteiros nao negativos com a seguinte propri-edade: dados dois quaisquer de seus elementos, o valor absoluto da diferenca entre elestambem pertence a S. Se d e o menor elemento positivo de S, prove que S consiste detodos os multiplos de d.

    Considere um elemento m qualquer de S. Pelo algoritmo da divisao, m = qd + r com0 r < d. Como todos os numeros m d,m 2d,m 3d, . . . ,m qd = r pertencema S e d e o menor elemento positivo de tal conjunto, devemos ter obrigatoriamente quer = 0. Sendo assim, podemos concluir que todos os elementos de S sao multiplos de d.Resta mostrarmos que todos os multiplos de d estao em S. Seja kd um multiplo positivoqualquer de d. Como S e infinito, existe um inteiro m S tal que m = qd > kd. Assimtodos os numeros m d,m 2d, . . . ,m (q k)d = kd estao em S.

    Definicao 2. Um inteiro a e um divisor comum de b e c se a | b e a | c. Se b e c nao saoambos nulos, denotaremos por mdc(b, c) o maximo divisor comum de b e c.

    Como um inteiro nao nulo possui apenas um numero finito de divisores, se b e c sao ambosnao nulos, o numero mdc(b, c) sempre existe, isto e, sempre esta bem definido.

    Lema 3. (Euclides) Se x 6= 0, mdc(x, y) = mdc(x, x+ y)

    Demonstracao. Seja d um divisor comum de x e y. Entao d | x+ y e consequentemente dtambem a um divisor comum de x e x+ y. Reciprocamente, se f e um divisor comum dex+ y e x, f tambem divide (x+ y) y = x e assim f e um divisor comum de x e y. Comoos conjuntos de divisores comuns dos dois pares de numeros mencionados sao os mesmos,o maior divisor comum tambem e o mesmo.

    Entao podemos calcular:

    mdc(123, 164) = mdc(123, 41) = mdc(41, 123) = mdc(41, 82) = mdc(41, 41) = 41.

  • POT 2012 - Teoria dos Numeros - Nvel 2 - Aula 3 - Samuel Feitosa

    Exemplo 4. Tres maquinas I, R, S imprimem pares de inteiros positivos em tickets. Paraa entrada (x, y), as maquinas I, R, S imprimem respectivamente (xy, y), (x+y, y), (y, x).Iniciando com o par (1, 2) podemos alcancar

    a) (819, 357)?

    b) (19, 79)?

    Para o item a), calculemos inicialmente mdc(819, 357):

    mdc(819, 357) = mdc(462, 357) = mdc(105, 357) = mdc(105, 252) = . . . = mdc(21, 21) = 21.

    Pelo Lema de Euclides, o mdc entre os dois numeros em um ticket nunca muda. Comomdc(1, 2) = 1 6= 21 = mdc(819, 357), nao podemos alcancar o par do item a).

    Para o item b), indiquemos com uma operacao de alguma das maquinas. Veja que:

    (2, 1)R (3, 1)

    S (1, 3)

    R (4, 3)

    R . . .

    R (19, 3)

    S (3, 19)

    R (22, 19)

    R (41, 19)

    R

    (60, 19)R (79, 19).

    Observacao 5. Procurar invariantes sempre e uma boa estrategia para comparar confi-guracoes diferentes envolvidas no problema. Confira o problema proposto 31.

    Definicao 6. Dizemos que dois inteiros p e q sao primos entre si ou relativamente primosse mdc(p, q) = 1. Dizemos ainda que a fracao p

    qe irredutvel se p e q sao relativamente

    primos.

    Exemplo 7. (IMO 1959) Prove que21n+ 4

    14n+ 3e irredutvel para todo numero natural n.

    Pelo lema de Euclides, mdc(21n+4, 14n+3) = mdc(7n+4, 14n+3) = mdc(7n+1, 7n+2) =mdc(7n+ 1, 1) = 1.

    O seguinte lema sera provado na proxima aula.

    Lema 8. (Propriedades do MDC) Seja mdc(a, b) = d, entao:

    i) Se k 6= 0, mdc(ka, kb) = kd.

    ii) mdc

    (a

    d,b

    d

    )= 1.

    iii) Se mdc(a, c) = 1, entao mdc(a, bc) = d.

    Exemplo 9. (Olimpada Inglesa) Se x e y sao inteiros tais que 2xy divide x2+y2x, proveque x e um quadrado perfeito

    2

  • POT 2012 - Teoria dos Numeros - Nvel 2 - Aula 3 - Samuel Feitosa

    Se d = mdc(x, y), entao x = da e y = db, com mdc(a, b) = 1. Do enunciado, temos:

    2abd2 | d2a2 + d2b2 da

    d2 | d2a2 + d2b2 da

    d2 | da

    d | a.

    Logo, a = dc, para algum c. Como x | y2, obtemos d2c | d2b2, ou seja, c|b2 e mdc(c, b2) = c.Usando que mdc(a, b) = 1 e que todo divisor comum de b e c tambem e um divisor comumde a e b, podemos concluir que mdc(c, b) = 1. Usando o item iii) do lema anterior,mdc(c, b2) = 1. Assim, c = 1 e x = d2c = d2.

    Exemplo 10. No planeta X, existem apenas dois tipos de notas de dinheiro: $5 e $78. Epossvel pagarmos exatamente $7 por alguma mercadoria? E se as notas fossem de $ 3 e $78?

    Veja que 2 78 31 5 = 1 e consequentemente 14 78 217 5 = 7. Basta darmos14 notas de de $ 78 para recebermos 217 notas de $ 5 como troco na compra de nossamercadoria. Usando as notas de $3 e $78 nao e possvel pois o dinheiro pago e recebidocomo troco por algo sempre e multiplo de 3 e 7 nao e multiplo de 3.

    Queremos estudar a versao mais geral desse exemplo. Quais sao os valores que podemospagar usando notas de $a e $b? Em particular, estaremos interessados em conhecer qual omenor valor que pode ser pago. Para responder essa pergunta, precisaremos do algoritmode Euclides:

    Teorema 11. (O Algoritmo de Euclides) Para os inteiros b e c > 0, aplique sucessivamenteo algoritmo da divisao para obter a serie de equacoes:

    b = cq1 + r1, 0 < r1 < c,

    c = r1q2 + r2, 0 < r2 < r1,

    r1 = r2q3 + r3, 0 < r3 < r2,...

    rj2 = rj1qj + rj , 0 < rj < rj1,

    rj1 = rjqj+1

    A sequencia de restos nao pode diminuir indefinidamente pois 0 ri < ri1 e existe apenasum numero finito de naturais menores que c. Assim, para algum j, obteremos rj+1 = 0.O maior divisor comum de b e c sera rj, ou seja, o ultimo resto nao nulo da sequencia dedivisoes acima.

    Demonstracao. Pelo Lema de Euclides,

    mdc(x+ qy, y) = mdc(x+ (q 1)y, y) = mdc(x+ (q 2)y, y) = . . . = mdc(x, y).

    3

  • POT 2012 - Teoria dos Numeros - Nvel 2 - Aula 3 - Samuel Feitosa

    Entao,mdc(b, c) = mdc(c, r1) = mdc(r1, r2) = . . . = mdc(rj1, rj) = rj .

    Exemplo 12. Calcule mdc(42823, 6409).

    Pelo Algoritmo de Euclides,

    42823 = 6 6409 + 4369

    6409 = 1 4369 + 2040

    4369 = 2 2040 + 289

    2040 = 7 289 + 17

    289 = 17 17.

    Portanto, mdc(42823, 6409) = 17.

    Podemos extrair mais informacoes do Algoritmo de Euclides. Para isso, iremos organizaras equacoes do exemplo acima de outra forma.

    Essencialmente, a equacao mdc(x+qy, y) = mdc(x, y) nos diz que podemos subtrair q vezesum numero de outro sem alterar o maximo divisor comum do par em questao. Realizandoesse procedimento sucessivas vezes, subtraindo o numero menor do maior, podemos obterpares com numeros cada vez menores ate que chegarmos em um par do tipo (d, d). Como omaximo divisor comum foi preservado ao longo dessas operacoes, d sera o maximo divisorcomum procurado. Iremos repetir o exemplo anterior registrando em cada operacao quantasvezes um numero e subtraido do outro. Isso sera feito atraves de dois pares de numerosauxiliares:

    (42823, 6409) | (1, 0)(0, 1)

    (4369, 6409) | (1,6)(0, 1)

    (4369, 2040) | (1,6)(1, 7)

    (289, 2040) | (3,20)(1, 7)

    (289, 17) | (3,20)(22, 147)

    (17, 17) | (355,2372)(22, 147)

    Da primeira linha para a segunda, como subtramos 6 vezes o numero 6409 de 42823,subtramos 6 vezes o par (0, 1) de (1, 0), obtendo: (1, 0) 6(0, 1) = (1,6). Se em umadada linha, temos:

    (x, x+ qy)) | (a, b)(c, d);

    entao, a proxima linha devera ser:

    (x, y) | (a, b)(c aq, d bq);

    4

  • POT 2012 - Teoria dos Numeros - Nvel 2 - Aula 3 - Samuel Feitosa

    porque representara a operacao de subtrairmos q vezes o primeiro numero do segundo. Vejaque o par (a, b) foi subtraido de (c, d) exatamente q vezes. Os numeros escritos nos ultimosdois pares representam os coeficientes dos numeros originais para cada numero do primeiropar. Por exemplo, analisando a linha:

    (289, 2040) | (3,20)(1, 7);

    obtemos que:

    289 = 3 42823 20 6409,

    2040 = 1 42823 + 7 6409.

    Em cada linha, essa propriedade e mantida pois a mesma subtracao que e realizada noprimeiro par tambem e realizada entre os dois ultimos pares. Analisando o ultimo par,podemos escrever 17 como combinacao de 42823 e 6409 de duas formas diferentes:

    17 = 22 42823 + 147 6409,

    17 = 355 42823 +2372 6409,

    Assim, se no planeta X tivessemos apenas notas de $42823 e $6409, poderamos compraralgo que custasse exatamente $17.

    Como conclusao da discussao anterior e do algoritmo de Euclides, podemos concluir que:

    Teorema 13. (Bachet-Be`zout) Se d = mdc(a, b), entao existem inteiros x e y tais queax+ by = d.

    De fato, a discussao anterior tambem nos mostra um algoritmo para encontrarmos x e y.Voltando a` discussao sobre o planeta X, podemos concluir em virtude do teorema anteriorque qualquer valor multiplo de d podera ser pago usando apenas as notas de $a e $b.Como todo valor pago, necessariamente e um multiplo do maximo divisor comum de a eb, descobrimos que o conjunto que procuravamos consiste precisamente do conjunto dosmultiplos de d.

    Observacao 14. (Para professores) A prova mais comum apresentada para o teorema an-terior baseia-se na analise do conjunto de todas as combinacoes lineares entre a e b e quasesempre se preocupa apenas com mostrar a existencia de x e y. Acreditamos que o algoritmopara encontrar x e y facilite o entendimento do teorema para os alunos mais jovens. Entre-tanto, frequentemente utilizemos apenas a parte da existencia descrita no enunciado. Alemdisso, preferimos discutir um exemplo numerico ao inves de formalizarmos uma prova esugerimos que o professor faca o mesmo com mais exemplos em aula.

    Exemplo 15. (Olimada Russa 1995) A sequencia a1, a2, ... de naturais satisfazmdc(ai, aj) =mdc(i, j) para todo i 6= j Prove que ai = i para todo i.

    5

  • POT 2012 - Teoria dos Numeros - Nvel 2 - Aula 3 - Samuel Feitosa

    Para qualquer inteiro n, mdc(a2n, an) = mdc(2n, n) = n, consequentemente n | an. Sejad um divisor qualquer de an diferente de n, entao d | mdc(ad, an). De mdc(ad, an) =mdc(d, n), podemos concluir que d | n. Sendo assim, todos os divisores de an que saodiferentes de n sao divisores de n. Como ja sabemos que an = nk, para algum k, naopodemos ter k > 1 pois nk nao divide n e assim conclumos que an = n.

    Exemplo 16. Mostre que mdc(2120 1, 2100 1) = 220 1.

    Pelo lema de Euclides,

    mdc(2120 1, 2100 1) = mdc(2120 1 220(2100 1), 2100 1),

    = mdc(220 1, 2100 1),

    = mdc(220 1, 2100 1 280(220 1)),

    = mdc(220 1, 280 1),

    = mdc(220 1, 280 1 260(220 1)),

    = mdc(220 1, 260 1),

    = mdc(220 1, 260 1 240(220 1)),

    = mdc(220 1, 240 1),

    = mdc(220 1, 240 1 220(220 1)),

    = mdc(220 1, 220 1) = 220 1.

    Exemplo 17. (Olimpada Russa 1964) Sejam x, y inteiros para os quais a fracao

    a =x2 + y2

    xy

    e inteira. Ache todos os possveis valores de a.

    A primeira estrategia e cancelar os fatores comuns com o objetivo de reduzir o problemaao caso em que x e y sao primos entre si. Seja d = mdc(x, y), com

    {x = d x0y = d y0

    , mdc(x0, y0) = 1,

    entao

    a =x2 + y2

    xy=

    x02 + y0

    2

    x0y0

    Nessa condicao, como x0 divide y20 e y0 divide x

    20, cada um deles e igual a 1, donde

    a =12 + 12

    1 1= 2.

    6

  • POT 2012 - Teoria dos Numeros - Nvel 2 - Aula 3 - Samuel Feitosa

    Definicao 18. Os inteiros a1, a2, . . . , an, todos diferentes de zero, possuem multiplo comumb se ai|b para i = 1, 2, . . . , n(note que a1a2 . . . an e um multiplo comum). O menor multiplocomum positivo para tal conjunto de inteiros e chamado de mnimo multiplo comum e seradenotado por mmc(a1, a2, . . . , an).

    Proposicao 19. Se a e b sao nao nulos, entao: mmc(a, b) mdc(a, b) = |ab|.

    (A prova desta proposicao tambem sera deixada para a proxima secao)

    Exemplo 20. (Olimpada Russa 1995) Sejam m e n interios positivos tais que:

    mmc(m,n) +mdc(m,n) = m+ n.

    Prove que um deles e divisvel pelo o outro.

    Se d = mdc(m,n), entao podemos escrever m = da e n = db. Pela proposicao anterior,

    mmc(m,n) =d2ab

    d= dab.

    Temos:

    mmc(m,n) +mdc(m,n)m n = 0

    dab+ d da db = 0

    ab+ 1 a b = 0

    (a 1)(b 1) = 0.

    Portanto, ou a = 1 e m | n ou entao b = 1 e n | m.

    Exemplo 21. (Torneio das Cidades 1998) Prove que, para quaisquer inteiros positivos a eb, a equacao mmc(a, a+ 5) = mmc(b, b+ 5) implica que a = b.

    Para o item a), como (a+ 5) a = 5, temos mdc(a, a+ 5) e igual a 1 ou 5. O mesmo valepara mdc(b, b+ 5). Pela proposicao anterior, temos:

    mmc(a, a+ 5) =a(a+ 5)

    mdc(a, a+ 5),

    mmc(b, b+ 5) =b(b+ 5)

    mdc(b, b+ 5).

    Suponha que mdc(a, a+5) = 5 e mdc(b, b+5) = 1, entao a(a+5) = 5b(b+5). Consequen-temente, a e multiplo de 5 e a(a+ 5) e multiplo de 25. Isso implica que b(b+ 5) tambem emultiplo de 5 e que mdc(b, b+ 5) > 1. Uma contradicao. Analogamente, nao podemos termdc(a, a+ 5) = 1 e mdc(b, b+ 5) = 5. Sendo assim, mdc(a, a+ 5) = mdc(b, b+ 5) e:

    a(a+ 5) b(b+ 5) = 0

    (a b)(a+ b+ 5) = 0.

    Como a+ b+ 5 > 0, conclumos que a = b.

    7

  • POT 2012 - Teoria dos Numeros - Nvel 2 - Aula 3 - Samuel Feitosa

    Exemplo 22. Uma maquina f executa operacoes sobre o conjunto de todos os pares deinteiros positivos. Para cada par de inteiros positivos, ela fornece um inteiro dado pelasregras:

    f(x, x) = x, f(x, y) = f(y, x), (x+ y)f(x, y) = yf(x, x+ y).

    Determine f(2012, 2012! + 1).

    Claramente mmc(x, x) = x e mmc(x, y) = mmc(y, x). Usando a proposicao anterior e olema de Euclides temos:

    (x+ y)mmc(x, y) = (x+ y)xy

    mdc(x, y)= y

    x(x+ y)

    mdc(x, x+ y)= y mmc(x, x+ y)

    Temos entao uma forte suspeita de que f = mmc. Seja S o conjunto de todos os pa-res de inteiros positivos (x, y) tais que f(x, y) 6= mmc(x, y), e seja (m,n) o par em Scom a soma m + n minima. Note que todo par da forma (n, n) nao esta em S poisf(n, n) = n = mmc(n, n). Assim, devemos ter m 6= n. Suponha sem perda de generalidadeque n > m. Portanto:

    nf(m,nm) = [m+ (nm)]f(m,nm)

    = (nm)f(m,m+ (nm))

    f(m,nm) =nm

    n f(m,n)

    Como o par (m,m n) nao esta em S, dado que a soma de seus elementos e menor quem+ n, temos:

    f(m,nm) = mmc(m,nm) nm

    n f(m,n) = (nm)mmc(m,m+ (nm))

    f(m,n) = mmc(m,n)

    Uma contradicao. Desse modo, S deve ser um conjunto vazio e f(x, y) = mmc(x, y)para todos os pares de inteiros positivos. Como 2012 | 2012!, mdc(2012, 2012! + 1) = 1 econsequentemente mmc(2012, 2012! + 1) = 2012(2012! + 1).

    Problemas Propostos

    Problema 23. Calcule:

    a) mdc(n, n2 + n+ 1).

    b) mdc(3 2012, 2 2012 + 1).

    8

  • POT 2012 - Teoria dos Numeros - Nvel 2 - Aula 3 - Samuel Feitosa

    c) mdc

    (240 + 1

    28 + 1, 28 + 1

    ).

    Problema 24. Encontre mdc(2n+ 13, n+ 7)

    Problema 25. Prove que a fracao 12n+130n+2 e irredutvel.

    Problema 26. Sejam a, b, c, d inteiros nao nulos tais que ad bc = 1. Prove que a+bc+d e uma

    fracao irredutvel.

    Problema 27. Mostre que mdc(am 1, an 1) = amdc(m,n) 1.

    Problema 28. Mostre que se mdc(a, b) = 1, entao:

    mdc(a+ b, a2 ab+ b2) = 1 ou 3

    Problema 29. Dado que mdc(a, 4) = 2, mdc(b, 4) = 2, prove que:

    mdc(a+ b, 4) = 4.

    Problema 30. Prove que, para todo natural n,

    mdc(n! + 1, (n+ 1)! + 1) = 1.

    Problema 31. No exemplo 4, determine todos os pares que podem ser obtidos comecando-secom o par (1, 2).

    Problema 32. Qual o maximo divisor comum do conjunto de numeros:

    {16n + 10n 1, n = 1, 2, 3 . . .}?

    Problema 33. A sequencia Fn de Farey e a sequencia de todos as fracoes irredutveisa

    bcom 0 a b n arranjados em ordem crescente.

    F1 = {0/1, 1/1}F2 = {0/1, 1/2, 1/1}F3 = {0/1, 1/3, 1/2, 2/3, 1/1}F4 = {0/1, 1/4, 1/3, 1/2, 2/3, 3/4, 1/1}F5 = {0/1, 1/5, 1/4, 1/3, 2/5, 1/2, 3/5, 2/3, 3/4, 4/5, 1/1}F6 = {0/1, 1/6, 1/5, 1/4, 1/3, 2/5, 1/2, 3/5, 2/3, 3/4, 4/5, 5/6, 1/1}

    Claramente, toda fracao ab< 1 com mdc(a, b) = 1, esta em algum Fn. Mostre que se m/n

    e m/n sao fracoes consecutivas em Fn temos |mn nm| = 1.

    Problema 34. (Resvista Quantum - Jornal Kvant) Todas as fracoes irredutveis cujos de-nominadores nao excedem 99 sao escritas em ordem crescente da esquerda para a direita:

    1

    99,1

    98, . . . ,

    a

    b,5

    8,c

    d, . . .

    Quais sao as fracoesa

    bec

    dem cada lado de

    5

    8?

    9

  • POT 2012 - Teoria dos Numeros - Nvel 2 - Aula 3 - Samuel Feitosa

    Problema 35. (OBM) Para cada inteiro positivo n > 1, prove que 1 + 12 +13 + . . .+

    1nnao

    e inteiro.

    Problema 36. Determine todas as solucoes em inteiros positivos para 1a+ 1

    b= 1

    c.

    Problema 37. Inteiros positivos a e b, relativamente primos, sao escolhidos de modo quea+ b

    a bseja tambem um inteiro positivo. Prove que pelo menos um dos numeros ab + 1 e

    4ab+ 1 e um quadrado perfeito.

    Problema 38. (IMO 1979) Sejam p, q numeros naturais primos entre si tais que:

    p

    q= 1

    1

    2+

    1

    3 . . .

    1

    1318+

    1

    1319.

    Prove que p e divisvel por 1979.

    Respostas, Dicas e Solucoes

    23. (a)

    mdc(n, n2 + n+ 1) = mdc(n, n2 + n+ 1 n(n+ 1)),

    = mdc(n, 1),

    = 1.

    (b)

    mdc(3 2012, 2 2012 + 1) = mdc(3 2012 (2 2012 + 1), 2 2012 + 1),

    = mdc(2012 1, 2 2012 + 1),

    = mdc(2012 1, 2 2012 + 1 2(2012 1)),

    = mdc(2012 1, 3),

    = mdc(2012 1 3 670, 3),

    = mdc(2, 3) = 1.

    Outra opcao seria observar que o mdc procurado deve dividir o numero 3(2 2012 + 1) 2(3 2012) = 3 e que 2 2012 + 1 nao e multiplo de 3.

    (c)

    mdc

    (240 + 1

    28 + 1, 28 + 1

    )= mdc

    (232 + 224 + 216 + 28 + 1, 28 + 1

    ),

    = mdc((232 1) + (224 + 1) + (216 1) + (28 + 1) + 1, 28 + 1

    ),

    = mdc(1, 28 + 1) = 1.

    10

  • POT 2012 - Teoria dos Numeros - Nvel 2 - Aula 3 - Samuel Feitosa

    24.

    mdc(2n+ 13, n+ 7) = mdc(2n+ 13 2(n+ 7), n+ 7),

    = mdc(2n+ 13 2(n+ 7), n+ 7),

    = mdc(1, n+ 7) = 1

    25.

    mdc(12n+ 1, 30n+ 2) = mdc(12n+ 1, 30n+ 2 2(12n+ 1)),

    = mdc(12n+ 1, 6n),

    = mdc(12n+ 1 2(6n), 6n),

    = mdc(1, 6n) = 1

    26. Seja f = mdc(a + b, c + d). Entao f | d(a + b) b(c + d) = 1 e consequentementef = 1.

    27. Veja que

    mdc(am 1, an 1) = mdc(amn 1 + (an 1)amn, an 1)

    = mdc(amn 1, an 1)

    O resultado segue aplicando o Algoritmo de Euclides aos expoentes.

    28. Seja f = mdc(a + b, a2 ab + b2). Entao f | (a + b)2 (a2 ab + b2) = 3ab. Semdc(f, a) > 0, devemos ter mdc(f, b) > 0 pois f | a + b. O mesmo argumento valepara mdc(f, b) > 0. Assim, mdc(f, a) = mdc(f, b) = 1. Portanto, f | 3.

    30. Pelo lema de Euclides,

    mdc(n! + 1, (n+ 1)! + 1) = mdc(n! + 1, (n+ 1)! + 1 (n+ 1)(n! + 1))

    = mdc(n! + 1,n)

    = mdc(n! + 1 n[(n 1)!],n) = 1

    34. Sejam l = mmc{1, 2, . . . , n} e ai = l/i. A soma considerada e

    a1 + a2 + . . .+ anl

    .

    Queremos analisar o expoente do fator 2 no numerador e no denominador. Seja k talque 2k n < 2k+1. Entao 2k||l e ai e par para todo i 6= 2

    k. Como a2k e mpar, segueque o numerador e mpar enquanto que o denominador e par. Consequentemente afracao anterior nao representa um inteiro.

    11

  • 36. Sejam d = mdc(a, b), a = dx, b = dy. Consequentemente mdc(x, y) = 1 e podemosescrever a equacao como:

    1

    a+

    1

    b=

    1

    c

    bc+ ac = ab

    dyc+ dxc = d2xy

    c(x+ y) = dxy

    Como mdc(xy, x+ y) = 1 pois mdc(x, y) = 1, devemos ter xy | c e consequentementec = xyk. Assim, d = k(x + y). O conjunto solucao e formado pelas triplas (a, b, c)onde (a, b, c) = (kx(x + y), ky(x + y), xyk) com mdc(x, y) = 1 e x, y e k inteirospositivos.

    38. Use a identidade de Catalao:

    11

    2+

    1

    3

    1

    4+ . . .

    1

    2n=

    1

    n+ 1+

    1

    n+ 2+ . . .+

    1

    2n

    Em seguida, agrupe os termos da forma1

    n+ i+

    1

    2n i+ 1e analise o numerador da

    fracao obtida.

    Referencias

    [1] S. B. Feitosa, B. Holanda, Y. Lima and C. T. Magalhaes, Treinamento Cone Sul 2008.Fortaleza, Ed. Realce, 2010.

    [2] D. Fomin, A. Kirichenko, Leningrad Mathematical Olympiads 1987-1991, MathProPress, Westford, MA, 1994.

    [3] D. Fomin, S. Genkin and I. Itenberg, Mathematical Circles, Mathematical Words, Vol.7, American Mathematical Society, Boston, MA, 1966.

    [4] I. Niven, H. S. Zuckerman, and H. L. Montgomery, An Introduction to the Theory ofNumbers.

  • Polos Olmpicos de TreinamentoCurso de Teoria dos Nmeros - Nvel 2Prof. Samuel Feitosa

    Aula 4

    Numeros Primos, MDC e MMC.

    Definicao 1. Um inteiro p > 1 e chamado numero primo se nao possui um divisor dsatisfazendo 1 < d < p. Se um inteiro a > 1 nao e primo, ele e chamado de numerocomposto. Um inteiro m e chamado de composto se |m| nao e primo.O proximo teorema nos diz que os primos sao as pecasfundamentais dos numeros inteiros:

    Teorema 2. Todo inteiro n, maior que 1, pode ser expresso como o produto de numeroprimo.

    Demonstracao. Se o inteiro n e um primo, entao ele mesmo e o produto de um unico fa-tor primo. Se o inteiro n nao e primo, existe uma decomposicao do tipo: n = n1n2 com1 < n1 < n e 1 < n2 < n. Repetindo o argumento para n1 e n2, podemos escrever n comoo produto de primos ou podemos obter parcelas menores escrevendo n como um produtode naturais. Como nao existe uma sucessao infinita de naturais cada vez menores, apos umnumero finito de operacoes desse tipo, poderemos escrever n como um produto de numerosprimos.

    Quantos numeros primos existem?

    Teorema 3. (Euclides) Existem infinitos numeros primos.

    Demonstracao. Suponha, por absurdo, que exita apenas uma quantidade finita de primos:p1, p2, . . . , pn. Considere o numero X = p1p1 . . . pn+1. Pelo teorema anterior, esse numerodeve ser o produto de alguns elementos do conjunto de todos os numeros primos. Entre-tanto, nenhum dos primos pi divide X.

    Exemplo 4. Existe um bloco de 1000 inteiros consecutivos nao contendo nenhum primo?

    Sim. Um exemplo e o conjunto 1001!+2, 1001!+3, . . . , 1001!+1001. Veja i | 1001!+ i paratodo i = 2, 3, . . . , 1001.

  • POT 2012 - Teoria dos Numeros - Nvel 2 - Aula 4 - Samuel Feitosa

    Exemplo 5. (Torneio das Cidades) Existe um bloco de 1000 inteiros consecutivos contendoapenas um primo?

    Para cada bloco de 1000 numeros consecutivos, contemos sua quantidade de numeros pri-mos. Por exemplo, no bloco 1, 2, 3, . . . , 1000, temos 168 numeros primos (mas so usaremoso fato de que existem mais de dois primos nesse bloco). Comparando os blocos consecuti-vos k + 1, k + 2, . . . , k + 1000 e k + 2, k + 3, . . . , k + 1001, ou o numero de numeros primosaumenta em uma unidade, ou fica constante ou diminui em uma unidade. Analisando to-dos os blocos consecutivos desde 1, 2, . . . , 1000 ate 1001! + 2, 1001! + 3, . . . , 1001! + 1001,o numero de numeros primos deve ser igual a` 1 em algum deles. Para ver isso, usare-mos um argumento de continuidade discreta: Comecando com o numero 168 e realizandoalteracoes de no maximo uma unidade na quantidade de primos em cada bloco, para che-garmos no numero 0, necessariamente deveremos passar pelo numero 1 em algum momento.

    Relembremos um importante resultado da aula passada:

    Teorema 6. (Bachet- Be`zout) Se d = mdc(a, b), entao existem inteiros x e y tais queax+ by = d.

    Proposicao 7. Sejam a, b e c inteiros positivos com a | bc e mdc(a, b) = 1. Entao, a | c.Demonstracao. Pelo teorema anterior, existem x e y inteiros tais que ax+ by = 1. Assim,acx+ bcy = c. Como a | acx e a | bcy, podemos concluir que a | c.

    Em particular, se p e um numero primo e p | ab, entao p | a ou p | b. Podemos usar essefato para garantir a unicidade em nosso primeiro teorema, obtendo o importante:

    Teorema 8. (Teorema Fundamental da Aritmetica) A fatoracao de qualquer inteiro n > 1,em fatores primos, e unica a menos da ordem dos fatores.

    Exemplo 9. (Russia 1995) E possvel colocarmos 1995 numeros naturais ao redor de umcrculo de modo que para quaisquer dois numeros vizinhos a razao entre o maior e o menorseja um numero primo?

    Nao, e impossvel. Suponha, por absurdo, que isso seja possvel e denotemos pora0, a1, . . . , a1995 = a0 tais inteiros. Entao, para k = 1, . . . , 1995,

    ak1ak

    e primo ou o in-verso de um primo. Suponha que a primeira situacao ocorra m vezes e a segunda ocorra1995 m vezes entre esses quocientes. Como o produto de todos os numeros da formaak1ak

    , para k = 1, . . . , 1995 e igual a` 1, podemos concluir que o produto de m primos deveser igual ao produto de 1995m primos. Em virtude da fatoracao unica, m = 1995m.Um absurdo pois 1995 e mpar.

    Proposicao 10. Se as fatoracoes em primos de n e m sao:

    n = p11 p22 . . . p

    kk ,

    m = p11 p22 . . . p

    kk .

    2

  • POT 2012 - Teoria dos Numeros - Nvel 2 - Aula 4 - Samuel Feitosa

    Entao, mdc(m,n) = p11 p22 . . . p

    kk e mmc(m,n) = p

    11 p

    22 . . . p

    kk , onde i e o menor dentre

    {i, i} e i e o maior dentre {i, i}.Proposicao 11. Se a e b sao inteiros positivos, mostre que mmc(a, b)mdc(a, b) = ab.

    Demonstracao. Basta usar a proposicao anterior e observar que:

    max{x, y}+min{x, y} = x+ y.

    Exemplo 12. (Torneio das Cidades 1998) E possvel que mmc(a, b) = mmc(a + c, b + c)para alguma conjunto {a, b, c} de inteiros positivos?Nao. Suponha que a+c e b+c possuem algum divisor primo p. Como p | mmc(a+c, b+c),caso existam tais inteiros, devemos ter que p | mmc(a, b). Assim, usando que pelo menosum dentre a e b e divisvel por p podemos concluir que c tambem e divisvel por p. Entao,podemos cancelar o fator p:

    mmc

    (a

    p,b

    p

    )=

    mmc(a, b)

    p=

    mmc(a+ c, b+ c)

    p= mmc

    (a+ c

    p,b+ c

    p

    ).

    Efetuando alguns cancelamentos, podemos supor entao que a+c e b+c nao possuem fatoresprimos em comum. Obtivemos um absurdo pois:

    mmc(a+ c, b+ c) = (a+ c)(b+ c) > ab mmc(a, b).

    Exemplo 13. (OCM 2005) Determinar os inteiros n > 2 que sao divisveis por todos osprimos menores que n.

    Como mdc(n, n 1) = 1, se n 1 possui algum fator primo, ele nao dividira n. Assim,n 1 < 2. Consequentemente nao existe tal inteiro.Exemplo 14. Mostre que n4 + n2 + 1 e composto para n >1.

    Veja que n4 + n2 + 1 = n4 + 2n2 + 1 n2 = (n2 + 1)2 n2 = (n2 + n + 1)(n2 n + 1).Para n > 1, n2 n+ 1 = n(n 1) + 1 > 1 e assim n4 + n2 + 1 e o produto de dois inteirosmaiores que 1.

    Exemplo 15. Mostre que n4 + 4n e composto para todo n > 1.

    Se n e par, certamente o numero em questao e divisvel por 4. Para o caso em que n eimpar, iremos usar a fatoracao:

    a4+4b4 = a4+4a2b2+4b4 4a2b2 = (a2+2b2) 4b2b2 = (a2 2ab+2b2)(a2+2ab+2b2).

    Para n da forma 4k + 1, faca a = n e b = 4k. Para n da forma 4k + 3, faca a = n eb = 22k+1.

    Exemplo 16. Se 2n+1 e um primo mpar para algum inteiro positivo n, prove que n e umapotencia de 2.

    3

  • POT 2012 - Teoria dos Numeros - Nvel 2 - Aula 4 - Samuel Feitosa

    Ja vimos que an 1 = (a 1)(an1 + an2 + . . .+ 1). Se n e impar,

    (a)n 1 = (a 1)((a)n1 + (a)n2 + . . .+ 1)an + 1 = (a+ 1)(an1 an2 + . . . a+ 1)

    Sendo assim, se n possusse algum divisor primo mpar p com n = pb, poderamos escrever:2n+1 = (a+1)(an1an2+ . . .a+1), onde a = 2b. Como an1an2+ . . .a+1 > 1,o numero 2n + 1 nao seria primo.

    Exemplo 17. Dados que p, p+ 10 e p+ 14 sao numeros primos, encontre p.

    Vamos analisar os possveis restos na divisao por 3 de p. Se p deixa resto 1, entao p + 14e um multiplo de 3 maior que 3 e consequentemente nao podera ser um numero primo. Seo resto e 2, entao p + 10 e um multiplo de 3 maior que 3 e tambem nao podera ser umnumero primo. Assim, o resto de p por 3 e 0 e consequentemente p = 3.

    Exemplo 18. (Austria-Polonia) Dados naturais n e a > 3 mpar, mostre que a2n 1 tem

    pelo menos n+ 1 divisores primos distintos.

    Usando a fatoracao da diferenca de quadrados, temos que:

    a2k 1 = (a2k1 + 1)(a2k2 + 1) . . . (a+ 1)(a 1).

    Assim, a2m

    + 1 | a2k 1 se k > m. Como a e impar, podemos concluir que:

    mdc(a2k

    + 1, a2m

    + 1) = mdc(a2k 1 + 2, a2m + 1) = mdc(2, a2m + 1) = 2.

    Sendo assim, na fatoracao:

    a2n 12n

    =(a2

    n1

    + 1)

    2

    (a2n2

    + 1)

    2. . .

    (a+ 1)

    2

    (a 1)2

    ,

    temos o produto de pelo menos n inteiros primos entre si e consequentemente seus fatores

    primos sao distintos. Para cada termo (a2i+1)2 , temos um fator primo pi+1 diferente de 2.

    Da, a2n 1 possui pelo menos n+ 1 fatores primos distintos, a saber, {2, p1, p2, . . . , pn}.

    Exemplo 19. (Rioplatense 1999) Sejam p1, p2, . . . , pk primos distintos. Considere todos osinteiros positivos que utilizam apenas esses primos (nao necessariamente todos) em suafatoracao em numeros primos, formando assim uma sequencia infinita

    a1 < a2 < < an < .

    Demonstre que, para cada natural c, existe um natural n tal que

    an+1 an > c.

    4

  • POT 2012 - Teoria dos Numeros - Nvel 2 - Aula 4 - Samuel Feitosa

    Suponha, por absurdo, que exista c > 0 tal que an+1 an c, n N. Isso significa queas diferencas entre os termos consecutivos de (an)n1 pertencem ao conjunto {1, 2, . . . , c},logo sao finitas. Sejam d1, d2, . . . , dr essas diferencas. Seja i o maior expoente de pi queaparece na fatoracao de todos os dj .

    Considere entao o numero M = p1+11 p2+12 pk+1k . E claro que M pertence a` sequencia,

    ou seja, M = an, para algum n. Vejamos quem sera an+1. Por hipotese, existe i tal quean+1 an = di. Como an+1 > an, existe um primo pj que divide an+1 com expoente maiorou igual a j + 1. Caso contrario,

    an < an+1 < p1+11 p

    2+12 pk+1k = an ,

    absurdo. Da, pj+1j |an p

    j+1j |di, novamente um absurdo, pela maximalidade de j .

    Logo, o conjunto de todas as diferencas nao pode ser finito e, portanto, dado qualquerc > 0, existe um natural n tal que an+1 an > c.

    Problemas Propostos

    Problema 20. Dado que p, 2p+ 1 e 4p2 + 1 sao numeros primos, encontre p.

    Problema 21. Dado o par de primos p e 8p2 + 1, encontre p.

    Problema 22. Dado o par de primos p e p2 + 2, prove que p3 + 2 tambem e um numeroprimo.

    Problema 23. Dado que p, 4p2 + 1 e 6p2 + 1 sao numeros primos, encontre p.

    Problema 24. Os numeros de Fermat sao os numeros da forma 22n

    + 1. Prove que oconjunto dos divisores primos dos termos da sequencia de Fermat e infinito.

    Problema 25. Mostre que todo inteiro n pode ser escrito de maneira unica na forma n = ab,onde a e um inteiro livre de quadrado e b e um quadrado perfeito. Um inteiro e dito livrede quadrado se nao e divisvel por nenhum quadrado perfeito maior que 1.

    Problema 26. Prove que todo primo maior que 3 e da forma 6k+1 ou 6k+5.

    Problema 27. Prove que todo inteiro da forma 3k+2 tem um fator primo da mesma forma.

    Problema 28. Prove que existem infinitos primos da forma 4k+3 e 6k +5.

    Problema 29. Prove que se n e composto, entao possui um fator primo p n.Problema 30. (OBM 1998) Sao dados 15 numeros naturais maiores que 1 e menores que1998 tais que dois quaisquer sao primos entre si. Mostre que pelo menos um desses 15numeros e primo.

    Problema 31. Mostre que n|(n-1)! para todo numero composto n.

    5

  • POT 2012 - Teoria dos Numeros - Nvel 2 - Aula 4 - Samuel Feitosa

    Problema 32. Suponha que n >1. Mostre que a soma dos inteiros dos inteiros positivosnao excedendo n divide o produto dos inteiros positivos nao excedendo n se, e somente se,n e composto.

    Exemplo 33. (Russia 1995) Encontre todos os primos p para os quais p2+11 tenha exata-mente seis divisores distintos, incluindo 1 e p2 + 11.

    Problema 34. (Irlanda 2002 ) Encontre todas as solucoes inteiras positivas de p(p+ 3) +q(q + 3) = n(n+ 3), onde p, q sao primos.

    Exemplo 35. Prove que qualquer quadrado perfeito positivo tem mais divisores que deixamresto 1 na divisao por 3 do que divisores que deixam resto 2 na divisao por 3.

    Dicas e Solucoes

    19. Analisemos o resto de p na divisao por 3. Se p deixar resto 1, o numero 2p + 1 seradivisvel por 3. Se p deixar resto 2, o numero 4p+ 1 sera divisvel por 3. Em ambosos casos, 2p+ 1, 4p+ 1 > 3 e obtemos assim um absurdo.

    20. Analisemos o resto de p na divisao por 3. Se p deixa resto 1 ou 2, p2 deixa resto 1e consequentemente 8p2 + 1 deixa resto 0 por 3 mas certamente e maior que 3. Umabsurdo, logo p = 3.

    21. Analisemos o resto na divisao por 3. Se p nao e multiplo de 3, p2 + 2 e divisvel por3 e maior que 3. Um absurdo, logo p = 3 e p3 + 2 = 29.

    22. Analise os restos na divisao por 5.

    23. Iremos usar a fatoracao do exemplo 17:

    22n 1 = (22n1 + 1)(22n2 + 1) . . . (2 + 1)(2 1).

    Assim, se k > m,

    mdc(22k

    + 1, 22m

    + 1) = mdc(22k 1 + 2, 22m + 1) = mdc(2, 22m + 1) = 1,

    produzindo que quaisquer dois numeros de Fermat distintos sao primos entre si e issonecessariamente implica que o conjunto de seus divisores primos e infinito.

    24. Analise os restos na divisao por 2 e 3.

    27. Tente imitar a prova de Euclides para a existencia de infinitos primos.

    29. Se n e composto, podemos escrever n = ab com 1 < a b

  • 31. Escreva n = ab e analise as aparicoes de a e b no produto (n 1) (n 2) . . . 2 1.33. Se p 6= 3, 3 | p2 + 11. Analogamente, se p 6= 2, 4 | p2 + 11. Assim, exceto nes-

    ses dois casos, 12 | p2 + 11 e podemos encontrar mais que 6 divisores distintos:{1, 2, 3, 4, 6, 12, p2+11}. Agora, teste p = 2 e p = 3 para verificar que p = 3 e a unicasolucao.

    34. Sejan = 3 p11 pnn q11 qmm

    a decomposicao de n em fatores primos, onde cada pi deixa resto 1 por 3 e cada qjdeixa resto 2 por 3. Entao

    n2 = 32 p211 p2nn q211 q2mm .

    Um divisor de n2 deixa resto 1 por 3 se e somente se possuir uma quantidade par deprimos qj , contados com repeticao. Mais especificamente, se e somente se a soma dosexpoentes de q1, . . . , qm for par. Assim, a quantidade de divisores dessa forma e iguala

    D1 = (21 + 1) (2n + 1)[1

    2(21 + 1)(22 + 1) (2m + 1) + 1

    ].

    Enquanto para se obter um divisor que deixe resto 2 por 3, precisamos de umaquantidade mpar de fatores primos da forma 3k+2. Assim, a quantidade de divisoresdessa forma e:

    D2 := (21 + 1)(22 + 1) (2n + 1)(1

    2(21 + 1)(22 + 1) (2m + 1)

    ).

    Da, segue facilmente que D1 > D2.

    Referencias

    [1] E. Carneiro, O. Campos and F. Paiva, Olimpadas Cearenses de Matematica 1981-2005(Nveis Junior e Senior), Ed. Realce, 2005.

    [2] S. B. Feitosa, B. Holanda, Y. Lima and C. T. Magalhaes, Treinamento Cone Sul 2008.Fortaleza, Ed. Realce, 2010.

    [3] D. Fomin, A. Kirichenko, Leningrad Mathematical Olympiads 1987-1991, MathProPress, Westford, MA, 1994.

    [4] D. Fomin, S. Genkin and I. Itenberg, Mathematical Circles, Mathematical Words, Vol.7, American Mathematical Society, Boston, MA, 1966.

    [5] I. Niven, H. S. Zuckerman, and H. L. Montgomery, An Introduction to the Theory ofNumbers.

  • Polos Olmpicos de TreinamentoCurso de Teoria dos Nmeros - Nvel 2Prof. Samuel Feitosa

    Aula 6

    Congruencias I

    Definicao 1. Dizemos que os inteiros a e b sao congrentes modulo m se eles deixam omesmo resto quando divididos por m. Denotaremos isso por a b (mod m).

    Por exemplo, 7 2 (mod 5), 9 3 (mod 6), 37 7 (mod 10) mas 5 6 3 (mod 4). Vejaque a b (mod m) se, e somente se, m | a b.

    Teorema 2. Se a b (mod m) e c d (mod m), entao:

    i) a+ c b+ d (mod m)

    ii) a c b d (mod m)

    iii) ka kb (mod m) k Z

    iv) ac bd (mod m)

    v) ak bk (mod m) k N

    vi) Se mdc(k,m) = d, entao ka kb (mod m) a b (mod m/d)

    Demonstracao. Sejam q1 e q2 tais que:

    a b = q1m

    c d = q2m

    Entao, (a+ c) (b+ d) = (q1 + q2)m. Logo, a+ c e b+ d deixam o mesmo resto por m econsequentemente a+ c b+d (mod m). Usando que a b (mod a)k bk e que m | a b,conclumos que m (mod a)k bk. Os demais itens serao deixados para o leitor.

    Em termos praticos, podemos realizar quase todas as operacoes elementares envolvendoigualdade de inteiros. Uma das diferencas cruciais e a operacao de divisao como mostra oultimo item do teorema anterior.

  • POT 2012 - Teoria dos Numeros - Nvel 2 - Aula 5 - Samuel Feitosa

    Exemplo 3. Calcule o resto de 4100 por 3.

    Como 4 1 (mod 3), temos 4100 1100 = 1 (mod 3).

    Exemplo 4. Calcule o resto de 4100 por 5.

    Como 4 1 (mod 5), temos 4100 (1)100 = 1 (mod 5).

    Exemplo 5. Calcule o resto de 4100 por 7.

    Voce deve ter percebido que encontrar relacoes do tipo a 1 (mod m) podem simplificarbastante o calculo de ak (mod m). Procuremos alguma relacao como essa para 4 e 7. Vejaque:

    40 1 (mod 7), 41 4 (mod 7), 42 2 (mod 7), 43 1 (mod 7).

    Assim,499 = (43)33 133 = 1 (mod 7).

    Como 43 1 (mod 7), os restos das potencias de 4 na divisao por 7 se repetem periodica-mente de 3 em 3 pois 43k+r 43k 4r 4r (mod 7).

    Exemplo 6. Qual o resto de 3636 + 4141 na divisao por 77?

    Inicialmente devemos perceber que existe uma relacao entre os numeros do problema: 36+41 = 77. Assim:

    36 41 (mod 77),

    (36)41 4141 (mod 77),

    3636(1 365) 3636 + 4141 (mod 77).

    Nosso proximo passo e encontrar o resto de 365 na divisao por 77. Como 36 1 (mod 7),365 1 (mod 7). Alem disso, 36 3 (mod 1)1 produzindo 365 35 1 (mod 1)1.Como mdc(7, 11) = 1 e ambos dividem 365 1, podemos concluir que 77 | 365 1. Logo,3636 + 4141 deixa resto 0 na divisao por 77.

    Exemplo 7. Prove que p2 1 e divisvel por 24 se p e um primo maior que 3.

    Se p e um primo maior que 3, p 1 (mod 3) e p 1 (mod 2). Da, p2 1 (mod 3).Alem disso, se p = 2k + 1, segue que p2 = 4k(k + 1) + 1 1 (mod 8) pois k(k + 1) e par.Como mdc(8, 3) = 1 e ambos dividem p2 1, segue que 24 | p2 1.

    Exemplo 8. (OCM-2001) Achar o menor natural n tal que 2001 e a soma dos quadradosde n inteiros

    Podemos concluir da solucao do problema anterior que todo todo inteiro mpar ao quadradodeixa resto 1 por 8. Usemos isso para estimar o valor de n. Sejam x1, x2, . . . , xn inteirosmpares tais que:

    x21 + x22 + . . . x

    2n = 2001.

    2

  • POT 2012 - Teoria dos Numeros - Nvel 2 - Aula 5 - Samuel Feitosa

    Analisando a congruencia modulo 8, obtemos:

    x21 + x22 + . . . x

    2n = 2001 (mod 8)

    1 + 1 + . . .+ 1 1 (mod 8)

    n 1 (mod 8)

    Como 2001 nao e quadrado perfeito, nao podemos ter n = 1. O proximo candidado paran seria 1 + 8 = 9. Se exibirmos um exemplo para n = 9, teremos achado o valor mnimo.Veja que:

    2001 = 432 + 112 + 52 + 12 + 12 + 12 + 12 + 12 + 12.

    Exemplo 9. (IMO) Seja s(n) a soma dos dgitos de n. Se N = 44444444, A = s(N) eB = s(A). Quanto vale s(B)?

    Pelo criterio de divisibilidade por 9, N A B (mod 9). Inicialmente calculemos oresto de N por 9. Como 4444 16 7 (mod 9), precisamos encontrar 74444 (mod 9).Seguindo os metodos dos primeiros exemplos, seria interessante encontrarmos um inteiro rtal que 7r 1 (mod 9). O menor inteiro positivo com essa propriedade e r = 3. Como4444 = 1481 3 + 1, temos:

    74444 714813+1 (73)1481 7 7 (mod 9).

    Nosso proximo passo e estimar o valor de s(B). Como N = 44444444 < 1054444, A =s(N) 5 4444 9 = 199980. Alem disso, B = s(A) 1 + 9 5 = 46 e s(B) 12. O unicointeiro menor ou igual a 12 com resto 7 por 9 e o proprio 7, da s(B) = 7.

    Exemplo 10. Prove que 11n+2 + 122n+1 e divisvel por 133 para qualquer natural n.

    Duas relacoes que podemos extrair dos numeros envolvidos sao: 14411 = 133 e 13312 =121. Assim:

    144 11 (mod 133),

    122 11 (mod 133),

    122n 11n (mod 133),

    122n+1 11n 12 (mod 133),

    122n+1 11n (121) + 133 11n (mod 133),

    122n+1 11n+2 (mod 133).

    Exemplo 11. Prove que n5 + 4n e divisvel por 5 para todo inteiro n

    Inicialmente note que n5 + 4n = n(n4 + 4). Se n 0 (mod 5), nao ha o que fazer. Sen 1 (mod 5), n4+4 1+4 = 0 (mod 5). Finalmente, se n 2 (mod 5), n2 4 1(mod 5) e consequentemente n4 + 4 1 + 4 = 0 (mod 5).

    Exemplo 12. Seja n > 6 um inteiro positivo tal que n 1 e n+ 1 sao primos. Mostre quen2(n2 + 16) e divisvel por 720. A recproca e verdadeira?

    3

  • POT 2012 - Teoria dos Numeros - Nvel 2 - Aula 5 - Samuel Feitosa

    Veja que n e da forma 6k, pois n 1 e n+ 1 sao primos maiores que 3, portanto da forma6k 1 e 6k + 1, respectivamente. Logo,

    n2(n2 + 16) = 144(9k4 + 4k2).

    Resta provar que 9k4+4k2 e um multiplo de 5. Vamos analisar a igualdade acima modulo5.

    i) Se k 0, 2 ou 3 (mod 5), temos 9k4 + 4k2 0 (mod 5);

    ii) Se k 1 (mod 5) n 1 (mod 5), temos n 1 0 (mod 5), um absurdo;

    iii) Se k 4 (mod 5) n 4 (mod 5), temos n+1 0 (mod 5), novamente um absurdo.

    Isso conclui a demonstracao. A recproca nao e verdadeira. Basta tomar, por exemplo,n = 90.

    Problemas Propostos

    Problema 13. Determine o resto de 220 1 na divisao por 41.

    Problema 14. Qual o resto de 12000 + 22000 + . . .+ 20002000 na divisao por 7?

    Problema 15. Qual o resto na divisao de 270 + 370 por 13?

    Problema 16. Qual o resto de 3200 por 100?

    Problema 17. (Estonia 2000) Determine todos os possveis restos da divisao do quadradode um numero primo com o 120 por 120.

    Problema 18. Qual o ultimo dgito de 777777?

    Exemplo 19. Prove que 22225555 + 55552222 e divisvel por 7.

    Problema 20. Prove que o numero n3 + 2n e divisvel por 3 para todo natural n.

    Problema 21. Prove que n2 + 1 nao e divisvel por 3 para nenhum n inteiro.

    Problema 22. Prove que n3 + 2 nao e divisvel por 9 para nenhum n inteiro.

    Problema 23. Prove que p2 q2 e divisvel por 24 se p e q sao primos maiores que 3.

    Problema 24. Prove que se 2n + 1 e 3n + 1 sao ambos quadrados perfeitos, entao n edivisvel por 40.

    Problema 25. Se n e mpar, prove que 7|22n+1 + 3n+2.

    Problema 26. Seja d(n) a soma dos dgitos de n. Suponha que n+ d(n) + d(d(n)) = 1995.Quais os possveis restos da divisao de n por 9?

    Problema 27. Prove que nao existem inteiros positivos x1, x2, . . . , x14 tais que:

    x41 + x42 + . . .+ x

    414 = 1599.

    4

  • POT 2012 - Teoria dos Numeros - Nvel 2 - Aula 5 - Samuel Feitosa

    Problema 28. Escreva uma unica congruencia que e equivalente ao par de congruenciasx 1 (mod 4) e x 2 (mod 3).

    Problema 29. Prove que 2015 1 e divisvel por 11 31 61

    Problema 30. (Alemanha 1997) Determine todos os primos p para os quais o sistema

    p+ 1 = 2x2

    p2 + 1 = 2y2

    tem uma solucao nos inteiros x, y.

    Problema 31. Mostre que se n divide um numero de Fibonacci entao ele dividira umainfinidade.

    Dicas e Solucoes

    13. Veja que

    25 = 32 9 (mod 41)

    210 81 1 (mod 42)

    220 1 (mod 41).

    Assim, o resto procurado e zero.

    14. Como i2000 (i + 7k)2000 (mod 7), podemos simplificar o problema calculando pri-meiramente o valor de:

    12000 + 22000 + 32000 + 42000 + 52000 + 62000 + 72000 (mod 7).

    Outra observacao importante que simplificara o calculo e perceber que 23 1 (mod 7).Assim,

    23k 1 (mod 7), 23k+1 2 (mod 7), e 23k+2 4 (mod 7).

    Usando isso e o fato de que 2000 e par, temos:

    12000 + 22000 + 32000 + 42000 + 52000 + 62000 + 72000

    12000 + 22000 + (4)2000 + 42000 + (2)2000 + (1)2000 + 02000

    1 + 4 + 2 + 2 + 4 + 1 + 0

    0 (mod 7).

    Dentre os primeiros 2000 naturais consecutivos, podemos formar 285 grupos de 7numeros consecutivos cuja soma e multipla de 7, em virtude da soma anterior. Oscinco numeros restantes possuem como resto na divisao por 7 o numero:

    19962000 + 19972000 + 19982000 + 19992000 + 20002000 1 + 4 + 2 + 2 + 4

    6 (mod 7).

    Assim, o resto da soma na divisao por 7 e 6.

    5

  • POT 2012 - Teoria dos Numeros - Nvel 2 - Aula 5 - Samuel Feitosa

    15. Inicialmente e interessante buscarmos alguma relacao entre os numeros envolvidos noproblema. Como 13 = 4 + 9, podemos escrever:

    9 4 (mod 13)

    935 (4)35 (mod 13)

    370 + 270 0 (mod 13).

    17. Use a fatoracao 120 = 3 5 23 e analise a congruencia modulo 3, 5 e 8 separadamente.

    18. Se n nao e multiplo de 3, sabemos que n2 1 (mod 3). Assim n2 + 2 0 (mod 3).Se n e multiplo de 3, n 0 (mod 3). Em qualquer caso, n(n2 + 2) 0 (mod 3).

    19. Basta repetir a analise do problema anterior

    20. Podemos montar uma tabela de congruencias na divisao por 9:

    n 0 1 2 3 4 5 6 7 8

    n3 0 1 8 0 1 8 0 1 8

    Como nenhum cubo perfeito diexa resto 7 na divisao por 9, n3 + 2 6 0 (mod 9).

    23. Proceda como no exemplo 7.

    25.

    22n+1 + 3n+2 4n 2 + 3n 9

    (3)n 2 + 3n 2

    0 (mod 7).

    26. Seja r o resto na divisao por 9 de n. Pelo criterio de divisibilidade por 9, temos:

    n+ d(n) + d(d(n)) 3r 1995 (mod 9).

    Assim, r 2 (mod 3) (Pela propriedade vi do teorema 2). Alem disso,

    n 1995

    d(n) 27 = d(1989)

    d(d(n)) 10 = d(19).

    Consequentemente, n 1995 d(n) d(d(n)) 1958. Basta procurarmos nosconjunto {1958, 1959, . . . , 1995} os inteiros que deixam resto 2 por 3 e que satisfazema equacao do problema. Nesse conjunto, apena o inteiro 1967 cumpre essas condicoes.

    27. Estudando a congruencia modulo 16, podemos mostrar que x4 0 ou 1 (mod 1)6.Assim, a soma

    x41 + x42 + . . .+ x

    414

    e congruente a um dos numeros do conjunto {0, 1, . . . , 14} m odulo 16 enquanto que1599 15 (mod 16). Um absurdo.

    6

  • 28. x 5 (mod 12).

    30. Suponha sem perda de generalidade que x, y 0. Como p + 1 e par, p 6= 2. Alemdisso,

    2x2 1 2y2 (mod p)

    e consequentente, usando que p e mpar, x y (mod p). Como x < y < p, temos

    p2 + 1 = 2(p x)2 = 2p2 4px+ p+ 1,

    de modo que p = 4x 1, 2x2 = 4x. Podemos concluir que x e 0 ou 2 e que a unicapossibilidade para p e p = 7.

    31. Em virtude da formula recursiva da sequencia de Fibonacci, e possvel mostrarmosque os restos de seus termos na divisao por qualquer numero formam uma sequenciaperiodica.

    Referencias

    [1] E. Carneiro, O. Campos and F. Paiva, Olimpadas Cearenses de Matematica 1981-2005(Nveis Junior e Senior), Ed. Realce, 2005.

    [2] S. B. Feitosa, B. Holanda, Y. Lima and C. T. Magalhaes, Treinamento Cone Sul 2008.Fortaleza, Ed. Realce, 2010.

    [3] D. Fomin, A. Kirichenko, Leningrad Mathematical Olympiads 1987-1991, MathProPress, Westford, MA, 1994.

    [4] D. Fomin, S. Genkin and I. Itenberg, Mathematical Circles, Mathematical Words, Vol.7, American Mathematical Society, Boston, MA, 1966.

    [5] I. Niven, H. S. Zuckerman, and H. L. Montgomery, An Introduction to the Theory ofNumbers.

  • Polos Olmpicos de TreinamentoCurso de Teoria dos Nmeros - Nvel 2Prof. Samuel Feitosa

    Aula 6

    Congruencias II

    Na aula de hoje, aprenderemos um dos teoremas mais importantes do curso: o pe-quenoteorema de Fermat. Comecaremos relembrando um resultado da aula passada:

    Lema 1. Se ka kb (mod m) e mdc(m, k) = 1, entao a b (mod m).

    Demonstracao. Como m | k(a b) e mdc(m, k) = 1, segue que m | a b.

    Teorema 2. (Teorema de Fermat) Seja p um primo. Se p nao divide a entao

    ap1 1 (mod p).

    Alem disso, para todo inteiro a, ap a (mod p)

    Demonstracao. Considere o conjunto de inteiros B = {a, 2a, 3a, . . . , (p 1)a} onde a e uminteiro satisfazendo mdc(a, p) = 1. Nenhum deles e divisvel por p e quaisquer dois delessao incongruentes modulo p, em virtude do lema anterior. Assim, o conjunto dos restosdos elementos de B coincide com o conjunto dos restos nao nulos na divisao por p, a saber,{1, 2, 3, . . . , p 1}. Portanto,

    a 2a 3a . . . (p 1)a 1 2 3 . . . (p 1) (mod p),

    ap1(p 1)! (p 1)! (mod p).

    Podemos cancelar o termo (p 1)! em ambos os lados pois mdc((p 1)!, p) = 1, concluindoassim a demonstracao do teorema.

    Exemplo 3. Prove quen5

    5+

    n3

    3+

    7n

    15e um inteiro para todo inteiro n.

    Primeiramente note quen5

    5+

    n3

    3+

    7n

    15=

    3n5 + 5n3 + 7n

    15. Como mdc(3, 5) = 1, basta

    mostrarmos que o numerador e mutiplo de 3 e 5. Pelo teorema de Fermat:

    3n5 + 5n3 + 7n 5n3 + 7n 5n+ 7n = 12n 0 (mod 3),

    3n5 + 5n3 + 7n 3n5 + 7n 3n+ 7n = 10n 0 (mod 5).

  • POT 2012 - Teoria dos Numeros - Nvel 2 - Aula 5 - Samuel Feitosa

    Problema 4. Mostre que n7 n (mod 42), n N

    Pelo teorema de Fermat,

    n7 n (mod 7)

    n7 (n3)2 n n2 n = n3 n (mod 3)

    n7 (n2)3 n n3 n = (n2)2 n2 n (mod 2)

    Como 2, 3 e 7 sao primos entre si, n7 n (mod 2 3 7 = 42).

    Exemplo 5. (Bulgaria 95) Encontre o numero de inteiros n > 1 para os quais o numeroa25 a e divisvel por n para cada inteiro a.

    Se n satisfaz o enunciado, p2(p primo) nao pode divid-lo, pois p25 p nao e divisvelpor p2. Assim, n e multiplo de primos diferentes. Os fatores primos de n sao fatores de225 2 = 2 32 5 7 13 17 241. Entretanto, n nao e divisvel por 17 e 241 pois 325 3(mod 17) e 325 32 (mod 241). Seguindo o exemplo anterior, podemos usar o teorema deFermat para mostrar que a25 a (mod p) para p {2, 3, 5, 7, 13}. Portanto, n deve serigual a um dos divisores de 2 3 5 7 13 diferente de 1. A quantidade de tais divisores e25 1 = 31.

    Exemplo 6. Prove que para cada primo p, a diferenca

    111 . . . 11222 . . . 22333 . . . 33 . . . 888 . . . 88999 . . . 99 123456789

    (onde cada digito esta escrito exatamente p vezes) e multiplo de p.

    Uma boa maneira de associar os numeros do problema com o teorema de Fermat e perceberque:

    111 . . . 11 p uns

    =10p 1

    9.

    Assim, podemos escrever o numero S = 111 . . . 11222 . . . 22333 . . . 33 . . . 888 . . . 88999 . . . 99como:

    S =10p 1

    9 108p + 2

    10p 1

    9 107p + . . . 9

    10p 1

    99S = (10p 1) 108p + 2 (10p 1) 107p + . . . 9 (10p 1)

    Para p = 2 ou p = 3, o resultado do enunciado segue dos criterios de divisibilidade por 2 e3. Podemos entao nos concentrar no caso p > 3. Nesse caso, e suficiente mostrarmos que9(S 123456789) e divisvel por p pois mdc(p, 9) = 1. Pelo teorema de Fermat:

    9S = (10p 1) 108p + 2 (10p 1) 107p + . . . 9 (10p 1)

    (10 1) 108 + 2 (10 1) 107 + . . . 9 + (10 1) (mod p)

    9 123456789 (mod p).

    2

  • POT 2012 - Teoria dos Numeros - Nvel 2 - Aula 5 - Samuel Feitosa

    Exemplo 7. Dado um primo p, prove que existem infinitos naturais n tais que p divide2n n.

    Se p = 2, n pode ser qualquer numero par. Suponha que p > 2. Considere (p 1)2k, peloteorema de Fermat temos:

    2(p1)2k

    (2p1)(p1)2k1

    1(p1)2k1

    = 1 (p 1)2k (mod p).

    Assim, para qualquer k, n = (p 1)2k satisfaz o problema.

    Lema 8. Se mdc(a,m) = 1 entao existe um inteiro x tal que

    ax 1 (mod m).

    Tal x e unico modulo m. Se mdc(a,m) > 1 entao nao existe tal x.

    Demonstracao. Pelo teorema de Bachet-Bezout, existem inteiros x e y tais que ax+my = 1.Analisando essa congruencia modulo m, obtemos ax 1 (mod m). Se y e outro inteiroque satisfaz a congruencia, temos ax ay (mod m). Pelo primeiro lema, x y (mod m).Se d = mdc(a,m) > 1, nao podemos ter d | m e m | ax 1 pois d ax 1.

    Teorema 9. (Teorema de Wilson) Se p e primo, entao

    (p 1)! 1 (mod p)

    Demonstracao. Em virtude do lema anterior, para cada a {2, 3, . . . , p 2}, existe umresto x {0, 1, 2, . . . , p1} tal que ax 1 (mod p). Se x = 1 ou x = p1, teramos a = 1ou p 1. Alem disso, nao podemos ter a = x pois os unicos restos que satisfazem a2 1(mod p) sao 1 e p 1 (Veja o problema 20). Com isso, podemos agrupar os numeros de{2, 3, . . . , p 2} em pares onde o produto deixa resto 1 por p, o que nos permite concluirque o produto de todos eles tambem deixa resto 1 por p. Logo,

    (p 1)! 1 (p 1) 1 (mod p).

    Exemplo 10. (Estonia 2000) Prove que nao e possvel dividir qualquer conjunto de 18inteiros consecutivos em dois conjuntos disjuntos A e B tais que o produtos dos elementosde A seja igual ao produto dos elementos de B.

    Suponha, por absurdo, que existam tais conjuntos. Considere o primo p = 19. Como oprodutos dos elementos de A e igual ao produtos dos elementos de B, se um dos conjuntoscontem um multiplo de 19, o outro necessariamente tambem contera. Como entre 18inteiros consecutivos nao existem dois multiplos de 19, nenhum dos conjuntos do problemacontem tais numeros. Seja x o resto na divisao por 19 dos produtos dos elementos de A.Calculemos entao o resto na divisao por 19 do produto de todos os 18 inteiros consecutivos:

    x x n(n+ 1)(n+ 2)(n+ 3) . . . (n+ 17)

    1 2 3 . . . 18

    1 (mod 19)(Pelo teorema de Wilson).

    Como x2 1 (mod 19), x18 (1)9 1 (mod 1)9. Isso contraria o teorema de Fermate obtemos um absurdo.

    3

  • POT 2012 - Teoria dos Numeros - Nvel 2 - Aula 5 - Samuel Feitosa

    Definicao 11. Um conjunto S e chamado de sistema completo de resduos modulo n, de-notado abreviadamente por scr, se para cada 0 i n 1, existe um elemento de s Stal que i s (mod n). Para qualquer a, o conjunto {a, a+ 1, a+ 2, . . . , a+ (n 1)} e umexemplo de scr.

    Exemplo 12. Se mdc(m, s) = 1, mostre que {t, t+ s, t+ 2s, . . . t+ (m 1)s} e um scr.

    Pelo primeiro lema, se t+ is t+ js (mod m), temos is js (mod m) e i j (mod m).Como i, j {0, 1, . . . ,m 1}, i = j. Isso nos diz que temos m inteiros que deixam restosdistintos na divisao porm. Como existem exatamentem restos na divisao porm, o conjuntoe um scr.

    Exemplo 13. Seja m um inteiro positivo par. Suponha que {a1, a1, . . . , am} e {b1, b2, . . . , bm}sao dois sistemas completos de resduso modulo m. Prove que

    S = {a1 + b1, a2 + b2, . . . , am + bm}

    nao e um sistema completo de resduos.

    Suponha que S seja um scr, entao:

    1 + 2 + . . .+m (a1 + b1) + (a2 + b2) + . . .+ (an + bn) (mod m)

    (a1 + a2 + . . .+ an) + (b1 + b2 + . . .+ bn)

    2(1 + 2 + . . .+ n)

    2(1 + 2 + . . .+m)

    Isso implica que m |m(m+ 1)

    2, ou seja,

    m+ 1

    2e inteiro. Um absurdo pois m e par.

    Exemplo 14. (Polonia 1997) Prove que a sequencia an definida por a1 = 1 e

    an = an1 + an2

    contem infinitos termos divisveis por 7.

    Uma maneira natural para mostrarmos que existem infinitos inteiros multiplos de 7 nasequencia e verificar que o aparecimento de um multiplo de 7 acarreta o aparecimento deoutro multiplo na sequencia com um ndice maior. Suponha que ak e multiplo de 7. Sejaa2k1 = s. Entao:

    a2k1 = s

    a2k = s+ ak s (mod 7)

    a2k+1 = a2k + ak s (mod 7)

    4

  • POT 2012 - Teoria dos Numeros - Nvel 2 - Aula 5 - Samuel Feitosa

    Ou seja, o aparecimento de um inteiro multiplo de 7 implica no aparecimento de 3 inteiroscom o mesmo resto por 7. Exploremos essa ideia mais uma vez.

    a4k3 = t

    a4k2 t+ a2k1 t+ s (mod 7)

    a4k1 t+ s+ a2k1 t+ 2s (mod 7)

    a4k t+ 2s+ a2k t+ 3s (mod 7)

    a4k+1 t+ 3s+ a2k t+ 4s (mod 7)

    a4k+2 t+ 4s+ a2k+1 t+ 5s (mod 7)

    a4k+3 t+ 5s+ a2k+2 t+ 6s (mod 7)

    Se s e multiplo de 7, ja teremos conseguido outro multiplo de 7 na sequencia. Em casocontrario, o conjunto {t, t+ s, t+ 2s, . . . , t+ 6s} e um scr e contera um multiplo de 7.

    Exemplo 15. Sejam x, y inteiros. Prove que 3x2 + 4y2 e 4x2 + 3y2 nao podem ser ambosquadrados perfeitos.

    Comecemos com um lema bastante util:

    Lema 16. Seja p um numero primo da forma 4k + 3. Entao

    p | m2 + n2 p | m e p | n.

    Facamos inicialmente a primeira implicacao. Se p m, entao mp1 1 (mod p), e datemos as equivalencias modulo p

    n2 m2

    (nmp2)2 (mp1)2

    1

    (nmp2)p1 (1)p1

    2

    (1)2k+1

    1,

    o que contraria o teorema de Fermat. Assim, p | m e p | n.

    A recproca e obvia. Voltando ao problema, suponha que existam w, z inteiros positivostais que

    3x2 + 4y2 = w2 e

    4x2 + 3y2 = z2.

    Entao 7x2+7y2 = w2+z2 (). Afirmamos que a equacao () nao possui solucao. Para isso,seja S o conjunto formado pelas solucoes inteiras (x, y, w, z) de (), e tome (a, b, c, d) S

    5

  • POT 2012 - Teoria dos Numeros - Nvel 2 - Aula 5 - Samuel Feitosa

    com c2 + d2 mnimo. Pelo lema, temos que 7|c e 7|d, e da c = 7c e d = 7d. Mas entaoa2 + b2 = 7c2 + 7d2 (c, d, a, b) S, com

    a2 + b2 < 7(a2 + b2) = c2 + d2,

    o que contraria a minimalidade de (a, b, c, d).

    Problemas Propostos

    Problema 17. Prove que se p e primo entao

    ap bp (mod p) ap bp (mod p2)

    Problema 18. Encontre os restos da divisoes de:

    a) 3003000 1 por 1001

    b) 7120 1 por 143

    Problema 19. Encontre o resto de 111 . . . 11 p1 uns

    por p, onde p e um primo maior que 5.

    Problema 20. Prove que se n e mpar, entao n5 n (mod 240).

    Problema 21. Sejam p e q primos distintos. Mostre que

    i) (a+ b)p ap + bp (mod p)

    ii) pq + qp p+ q (mod pq)

    iii)

    pq + pq

    pq

    e par se p, q 6= 2.

    Problema 22. Mostre que se p e primo e a2 b2 (mod p), entao a b (mod p).

    Problema 23. Encontre os ultimos tres dgitos de 79999

    Problema 24. Prove que 2015 1 e divisvel por 11 31 61

    Problema 25. Sejam {a1, a2, ..., a101} e {b1, b2, ..., b101} sistemas completos de resduosmodulo 101. Pode {a1b1, a2b2, ..., a101b101} ser um sistema completo de resduos modulo101?

    Problema 26. (Balcanica 2003) Existe um conjunto B de 4004 inteiros positivos tal que,para cada subconjunto A de B com 2003 elementos, a soma dos elementos em A nao edivisvel por 2003?

    Problema 27. Para um inteiro mpar n > 1, seja S o conjunto de inteiros x,1 x n,tal que ambos x e x + 1 sao relativamente primos com n. Mostre que o produto de todosos elementos de S deixa resto 1 na divisao por n.

    6

  • POT 2012 - Teoria dos Numeros - Nvel 2 - Aula 5 - Samuel Feitosa

    Problema 28. Sejam n um inteiro positivo maior que 1 e p um primo positivo tal que ndivide p 1 e p divide n3 1. Mostre que 4p 3 e um quadrado perfeito.

    Dicas e Solucoes

    17. Pelo teorema de Fermat, a ap bp b (mod p). Assim,

    ap1 + ap2b+ . . .+ abp2 + bp1 ap1 + ap1 + . . .+ ap1

    pap1

    0 (mod p)

    Como a b 0 (mod p), temos:

    ap bp = (a b)(ap1 + ap2b+ . . .+ abp2 + bp1) 0 (mod p2)

    19. Veja que:

    111 . . . 11 p1 uns

    =999 . . . 99

    9

    =10p1 1

    9

    Pelo teorema de Fermat, o numerador 10p1 1 e divisvel por p visto que p 6= 5.Alem disso, usando que p 6= 2 e 3, segue que 10

    p11

    9 tambem e multiplo de p.

    20. Proceda como no exemplo 20.

    21. i)Pelo teorema de Fermat:

    (a+ b)p a+ b

    ap + bp (mod p).

    ii) Pelo teorema de Fermat,

    pq + qp 0 + q p+ q (mod p)

    pq + qp p+ 0 p+ q (mod q)

    22. Veja que (a b)(a+ b) 0 (mod p) e assim a b 0 (mod p) ou a+ b 0 (mod p).

    25. Suponha, por abusurdo, que seja possvel. Sejam ai e bj tais que ai bj 0(mod 101). Se i 6= j, o conjunto {a1b1, a2b2, ..., a101b101} teria dois inteiros com resto

    7

  • POT 2012 - Teoria dos Numeros - Nvel 2 - Aula 5 - Samuel Feitosa

    0 na divisao por p e nao poderia ser um scr. Suponha sem perda de generalidade quei = j = 101, entao:

    100! (a1b1)(a2b2) . . . (a100b100)

    (a1a2 . . . a100)(b1b2 . . . b100)

    (100!)(100!)

    (100!)2 (mod 101)

    Assim, 100! 1 (mod 101). Isso contradiz o teorema de Wilson.

    26. Sim. Um exemplo de tal conjunto e a uniao de um conjunto de 2002 inteiros positivosque deixem resto 0 com outro conjunto composto por 2002 inteiros que deixem resto1 por 2003.

    8

  • Polos Olmpicos de TreinamentoCurso de Teoria dos Nmeros - Nvel 2Prof. Samuel Feitosa

    Aula 7

    Aula de Revisao e Aprofundamento

    Observacao 1. E recomendavel que o professor instigue seus alunos a pensarem nos pro-blemas abaixo antes de resolve-los na aula.

    Exemplo 2. (ASHME 1990) Para quantos inteiros N entre 1 e 1990 a fracaoN2 + 7

    N + 4nao

    e irredutvel?

    Seja d = (N +4, N2 +7). Um boa estrategia e procurar um multiplo de N +4 proximo deN2+7 pois assim conseguiremos estimar d. Usando a diferenca de quadrados, d | N216 econsequentemente d | N2+7(N216) = 23. Como 23 e primo, a fracao nao sera irredutvelapenas quando d = 23. Para isso acontecer, basta que 23 | N+4 pois N2+7 = N216+23.O maior multiplo de 23 menor que 1990 e 1978 = 23 86 e 1990+ 4 < 23 87. Sendo assim,a quantidade de inteiros procurada e 86.

    Exemplo 3. Dados os primos p e q satisfazendo:

    q | p2 + 1 e p | q2 1.

    Prove que o numero p+ q + 1 e composto.

    Como p | q21 = (q+1)(q1), temos que p | q+1 ou p | q1. No primeiro caso, p+q+1e um multiplo de p. No segundo caso, podemos escrever q 1 = pk para algum natural k.Usando que q | p2 + 1, conclumos que q | p2 + 1 (pk + 1) = p(p k). Como p e q naopodem ser primos iguais, q | p k. Temos tres casos a considerar:

    1. p > k. Entao:

    p k kp+ 1,(p+ 1)(1 k) 2

  • 2. p < k. Entao:

    k p kp+ 1,(k + 1)(1 p) 2

    3. p = k e q = p2+1. Como o unico primo par e 2, segue que p = 2, q = 5 e p+q+1 = 8.

    Os dois primeiros casos conduzem a um absurdo. Logo, ou p+ q + 1 e multiplo de p ou eigual a` 8.

    Exemplo 4. (AIME 1985) Os numeros da sequencia

    101, 104, 109, 116, . . .

    sao da forma an = 100 + n2, onde n = 1, 2, 3, . . . Para cada n, seja dn o maximo divisor

    comum de an e an+1. Encontre o valor maximo de dn quando n varia sobre todo o conjuntodos inteiros positivos.

    Uma boa estrategia e buscar alguma fatoracao que nos permita identificar fatores comunsentre os termos da sequencia. Um termo generico da sequencia possui a forma an =k + n2. Sendo assim, a2k = k(4k + 1),a2k+1 = (k + 1)(4k + 1) e consequentementemdc(a2k, a2k+1) = 4k + 1. Nosso proximo passo sera mostrar que realmente esse e o valormaximo. Considere dois termos genericos a = an = k + n

    2, b = an+1 = k + (n+ 1)2 e seja

    d = mdc(a, b). Usando que d | b a = 2n+1, obtemos d | (2n+1)(b a) = 4n2 1. Comod | 4a = 4n2 + 4k, segue que d | 4n2 + 4k (4n2 1) = 4k + 1. Assim, 4k + 1 e realmenteo maior valor possvel entre os termos da sequencia dn.

    Exemplo 5. Prove que para qualquer inteiro n > 1, o numero n5+n4+1 nao e um numeroprimo.

    Considere a fatoracao:

    n5 + n4 + 1 = n5 + n4 + n3 n3 n2 n+ n2 + n+ 1= n3(n2 + n+ 1) n(n2 + n+ 1) + (n2 + n+ 1)= (n2 + n+ 1)(n3 n+ 1)

    Como n > 1, n3 n+ 1 > 1 e obtemos assim o produto de dois inteiros maiores que 1.Exemplo 6. (Olimpada Grega) Encontre todos os inteiros n para os quais 54 +55 +5n eum quadrado perfeito.

    Como 54 + 55 = 2500, queremos encontrar m e n tais que:5n = m2 2500 = (m 50)(m+ 50).

    Isto implica que m+ 50 = 5j e m 50 = 5i, com i < j. Assim,100 = 5i(5ji 1).

    Usando a fatoracao em primos de 100, encontramos que i = 2 e ji = 1. Portanto, m = 75e n = 5.

    2

  • Exemplo 7. (Irlanda) Sejam p um numero primo, a e n e inteiros positivos. Prove que se

    2p + 3p = an,

    entao n = 1.

    Se p = 2, claramente a = 13 e n = 1. Se p > 2, p e mpar e 5 | 2p + 3p. Consequentemente5 divide a. Se fosse n > 1, 25 | an e teramos:

    0 an

    5

    2p + 3p

    5= 2p1 2p2 3 + . . .+ 2 3p2 + 3p1 2p1 + 2p1 + . . .+ 2p1 p2p1 (mod 5)

    A unica possbilidade e termos p = 5. Entretanto, 25 +35 nao e uma potencia perfeita naotrivial. Logo, n = 1.

    Exemplo 8. Um inteiro n > 1 tem a seguinte propriedade: para todo divisor positivo d den, d+ 1 e um divisor de n+ 1. Prove que n e primo.

    Seja p o menor fator primo de n e seja d =n

    p. Entao,

    np+ p

    n+ p=

    p(n+ 1)

    p(d+ 1)=

    n+ 1

    d+ 1

    e um numero inteiro. Como n + p tambem divide p(n + p), podemos concluir que n + p |(np+ p2) (np+ p) = p2 p. Em particular,

    n+ p p2 pn p2 2pn p2 2p+ 1 = (p 1)2n < p2

    d < p.

    Em virtude da minimalidade de p, d nao possui fatores primos e consequentemente n = p.

    Exemplo 9. (Olimpada Russa) Mostre que qualquer natural pode ser escrito como a dife-renca de dois numeros naturais tendo o mesmo numero de fatores primos.

    Se n e par, podemos escreve-lo como 2n n e e facil verificar que 2n e n possuem omesmo numero de fatores primos. Seja d o menor primo mpar que nao divide n. Escrevan = dn (d 1)n. O termo dn contem exatamente um primo a mais que n. Pela escolhade d, todos os outros fatores primos diferentes 2 do numero d1 sao divisores de n e assim(d 1)n tambem contem extamente um primo a mais que n, a saber, o primo 2.

    3

  • Exemplo 10. Os numeros naturais a e b sao tais que

    a+ 1

    b+

    b+ 1

    a

    e um numero inteiro. Mostre que o maximo divisor comun de a e b nao e maior quea+ b.

    Seja d = mdc(a, b), com a = md e b = nd. Entao:

    md+ 1

    nd+

    nd+ 1

    md=

    m2d+m+ n2d+ n

    mnd

    e um inteiro. Em pa